Hypothesis Testing (cont...)

Hypothesis testing, the null and alternative hypothesis.

In order to undertake hypothesis testing you need to express your research hypothesis as a null and alternative hypothesis. The null hypothesis and alternative hypothesis are statements regarding the differences or effects that occur in the population. You will use your sample to test which statement (i.e., the null hypothesis or alternative hypothesis) is most likely (although technically, you test the evidence against the null hypothesis). So, with respect to our teaching example, the null and alternative hypothesis will reflect statements about all statistics students on graduate management courses.

The null hypothesis is essentially the "devil's advocate" position. That is, it assumes that whatever you are trying to prove did not happen ( hint: it usually states that something equals zero). For example, the two different teaching methods did not result in different exam performances (i.e., zero difference). Another example might be that there is no relationship between anxiety and athletic performance (i.e., the slope is zero). The alternative hypothesis states the opposite and is usually the hypothesis you are trying to prove (e.g., the two different teaching methods did result in different exam performances). Initially, you can state these hypotheses in more general terms (e.g., using terms like "effect", "relationship", etc.), as shown below for the teaching methods example:

Null Hypotheses (H ): Undertaking seminar classes has no effect on students' performance.
Alternative Hypothesis (H ): Undertaking seminar class has a positive effect on students' performance.

Depending on how you want to "summarize" the exam performances will determine how you might want to write a more specific null and alternative hypothesis. For example, you could compare the mean exam performance of each group (i.e., the "seminar" group and the "lectures-only" group). This is what we will demonstrate here, but other options include comparing the distributions , medians , amongst other things. As such, we can state:

Null Hypotheses (H ): The mean exam mark for the "seminar" and "lecture-only" teaching methods is the same in the population.
Alternative Hypothesis (H ): The mean exam mark for the "seminar" and "lecture-only" teaching methods is not the same in the population.

Now that you have identified the null and alternative hypotheses, you need to find evidence and develop a strategy for declaring your "support" for either the null or alternative hypothesis. We can do this using some statistical theory and some arbitrary cut-off points. Both these issues are dealt with next.

Significance levels

The level of statistical significance is often expressed as the so-called p -value . Depending on the statistical test you have chosen, you will calculate a probability (i.e., the p -value) of observing your sample results (or more extreme) given that the null hypothesis is true . Another way of phrasing this is to consider the probability that a difference in a mean score (or other statistic) could have arisen based on the assumption that there really is no difference. Let us consider this statement with respect to our example where we are interested in the difference in mean exam performance between two different teaching methods. If there really is no difference between the two teaching methods in the population (i.e., given that the null hypothesis is true), how likely would it be to see a difference in the mean exam performance between the two teaching methods as large as (or larger than) that which has been observed in your sample?

So, you might get a p -value such as 0.03 (i.e., p = .03). This means that there is a 3% chance of finding a difference as large as (or larger than) the one in your study given that the null hypothesis is true. However, you want to know whether this is "statistically significant". Typically, if there was a 5% or less chance (5 times in 100 or less) that the difference in the mean exam performance between the two teaching methods (or whatever statistic you are using) is as different as observed given the null hypothesis is true, you would reject the null hypothesis and accept the alternative hypothesis. Alternately, if the chance was greater than 5% (5 times in 100 or more), you would fail to reject the null hypothesis and would not accept the alternative hypothesis. As such, in this example where p = .03, we would reject the null hypothesis and accept the alternative hypothesis. We reject it because at a significance level of 0.03 (i.e., less than a 5% chance), the result we obtained could happen too frequently for us to be confident that it was the two teaching methods that had an effect on exam performance.

Whilst there is relatively little justification why a significance level of 0.05 is used rather than 0.01 or 0.10, for example, it is widely used in academic research. However, if you want to be particularly confident in your results, you can set a more stringent level of 0.01 (a 1% chance or less; 1 in 100 chance or less).

Testimonials

One- and two-tailed predictions

When considering whether we reject the null hypothesis and accept the alternative hypothesis, we need to consider the direction of the alternative hypothesis statement. For example, the alternative hypothesis that was stated earlier is:

Alternative Hypothesis (H ): Undertaking seminar classes has a positive effect on students' performance.

The alternative hypothesis tells us two things. First, what predictions did we make about the effect of the independent variable(s) on the dependent variable(s)? Second, what was the predicted direction of this effect? Let's use our example to highlight these two points.

Sarah predicted that her teaching method (independent variable: teaching method), whereby she not only required her students to attend lectures, but also seminars, would have a positive effect (that is, increased) students' performance (dependent variable: exam marks). If an alternative hypothesis has a direction (and this is how you want to test it), the hypothesis is one-tailed. That is, it predicts direction of the effect. If the alternative hypothesis has stated that the effect was expected to be negative, this is also a one-tailed hypothesis.

Alternatively, a two-tailed prediction means that we do not make a choice over the direction that the effect of the experiment takes. Rather, it simply implies that the effect could be negative or positive. If Sarah had made a two-tailed prediction, the alternative hypothesis might have been:

Alternative Hypothesis (H ): Undertaking seminar classes has an effect on students' performance.

In other words, we simply take out the word "positive", which implies the direction of our effect. In our example, making a two-tailed prediction may seem strange. After all, it would be logical to expect that "extra" tuition (going to seminar classes as well as lectures) would either have a positive effect on students' performance or no effect at all, but certainly not a negative effect. However, this is just our opinion (and hope) and certainly does not mean that we will get the effect we expect. Generally speaking, making a one-tail prediction (i.e., and testing for it this way) is frowned upon as it usually reflects the hope of a researcher rather than any certainty that it will happen. Notable exceptions to this rule are when there is only one possible way in which a change could occur. This can happen, for example, when biological activity/presence in measured. That is, a protein might be "dormant" and the stimulus you are using can only possibly "wake it up" (i.e., it cannot possibly reduce the activity of a "dormant" protein). In addition, for some statistical tests, one-tailed tests are not possible.

Rejecting or failing to reject the null hypothesis

Let's return finally to the question of whether we reject or fail to reject the null hypothesis.

If our statistical analysis shows that the significance level is below the cut-off value we have set (e.g., either 0.05 or 0.01), we reject the null hypothesis and accept the alternative hypothesis. Alternatively, if the significance level is above the cut-off value, we fail to reject the null hypothesis and cannot accept the alternative hypothesis. You should note that you cannot accept the null hypothesis, but only find evidence against it.

U.S. flag

An official website of the United States government

The .gov means it’s official. Federal government websites often end in .gov or .mil. Before sharing sensitive information, make sure you’re on a federal government site.

The site is secure. The https:// ensures that you are connecting to the official website and that any information you provide is encrypted and transmitted securely.

  • Publications
  • Account settings

The PMC website is updating on October 15, 2024. Learn More or Try it out now .

  • Advanced Search
  • Journal List
  • HHS Author Manuscripts

Logo of nihpa

BETTER STATISTICS FOR BETTER DECISIONS: REJECTING NULL HYPOTHESES STATISTICAL TESTS IN FAVOR OF REPLICATION STATISTICS

Despite being under challenge for the past 50 years, null hypothesis significance testing (NHST) remains dominant in the scientific field for want of viable alternatives. NHST, along with its significance level p , is inadequate for most of the uses to which it is put, a flaw that is of particular interest to educational practitioners who too often must use it to sanctify their research. In this article, we review the failure of NHST and propose p rep , the probability of replicating an effect, as a more useful statistic for evaluating research and aiding practical decision making.

Statistics can address three different types of questions ( Royall, 1997 ):

  • What should I believe?
  • How should I evaluate this evidence?
  • What should I do?

The first two are of great importance to scientists: Finding the answers to these questions defines their praxis. The last question, on which we focus here, is of greater relevance to practitioners who must deal with decisions that have practical consequences. In its simplest form, a decision is a choice between two alternative courses of action. All other things being equal, optimal decisions favor courses of action that are expected to yield higher returns (e.g., better indices of class attendance) and are less costly to implement over those that are expected to yield lower returns and cost more. Choices with dominant alternatives are trivial; it is when costs and expected returns covary in the same direction that practical choices may become dilemmas, invoking the aid of decision committees and statisticians. Costly actions must be justified by their returns exceeding some minimum standard of expected improvement. Once that minimum improvement is defined, researchers produce data from which statisticians, in turn, are expected to determine whether the minimum improvement is real or not. Null hypothesis significance tests (NHST) are the conventional tool for making these evaluations. The privileged status of NHST is most clearly reflected in its prevalence as a diagnostic tool in the psychological and educational literature and in its entrenchment in the statistical training of education and psychology professionals. We will illustrate its use by applying the NHST routine to the solution of a practical binary-choice situation and demonstrate its inadequacy in informing a decision in that scenario. We argue that the probability of obtaining a minimum cost-effective return is more informative than arbitrary decisions about statistical significance and provide the rationale and algorithm for its estimation.

The Null Hypothesis Significance Testing Routine

Imagine that you are asked to evaluate a method for teaching English as a second language (ESL). How would you decide whether this new method for teaching ESL is better than the traditional one? First, you would collect data from two groups of students being taught ESL, matched as closely as possible on all potentially relevant variables, one using the old teaching method (Group OLD), and the other the new method (Group NEW). At the end of the course, you would obtain a validated measure of ESL performance (e.g., CBT-TOEFL) from each student; this score, or the student’s change score, is the measure of the return yielded by each teaching method. These results are then entered as two columns of numbers in an SPSS spreadsheet; what to do with them depends on the research question. With certain assumptions, a specific NHST procedure such as a one- or two-tailed t test would allow a researcher to determine the probability p that the difference in mean TOEFL scores between Group OLD and Group NEW would be obtained, given that those scores are sampled from the same population distribution—that is, given that the intervention did not separate the groups into two subpopulations with different labels such as “speaks English better.” This difference is often normalized and reported as effect size , measured as

where s POOLED is the pooled standard deviation of performances. The null hypothesis , which assumes that scores in both groups are samples of the same population distribution, is H 0 : μ NEW = μ OLD . A p value indicates the probability of d ′ given H 0 , or p ( d& prime;| H 0 ).

We specified a simple binary choice and followed the traditional NHST steps to reach a decision—but instead we obtained a p value. What decision should be made based on a particular p value? If the p value is below a criterion α , say .01, we would conclude that the difference between groups is “significant,” and would probably decide to adopt the new method for teaching ESL, other things being equal. This decision procedure, however, presents two problems.

Problem 1: Confidence

If other things are truly equal, a significance test is unnecessary. Regardless of the p value obtained, the method that yields higher mean TOEFL scores should be adopted, given that there is no differential cost in implementing one or the other. Although the population means may be predicted with more or less accuracy by the sample means, a predicted improvement should be pursued, assuming there is no cost for changing techniques. The triviality of this choice vanishes when other things are not equal. In the ESL example, the better teaching method must have expected returns that are high enough to offset the cost of its implementation (i.e., retraining, acquisition of new material). How should this minimum difference be factored into a decision, given p and a d& prime; values? The usual strategy is to focus on the effect size d ′: If d ′ is greater than a minimum criterion (e.g., d ′ > 0.3) the better alternative is adopted, regardless of cost—otherwise it is not. However, d ′ is an estimate of the difference between groups: How certain can one be of that estimate? More generally, what is the certainty that, given an obtained difference d ′ = 0.3, further tests would demonstrate any difference in the same direction (i.e., d ′ > 0)?

One conventional solution is to obtain a confidence interval (CI) of the difference of treatment means. A CI provides a range of treatment differences within which the population difference is expected 100(1 > α )% of the time. That is, if 1000 comparisons between ESL teaching methods were made, and if in each comparison a 95% CI was drawn around each mean difference (i.e., α = .05), we would expect approximately 950 of those intervals to include the population difference between ESL methods. Unfortunately, a single CI is frequently misinterpreted as the range within which 950 of the next 1000 mean differences would fall. The difference between these two interpretations is illustrated in Figure 1 . Data points are d ′ values obtained from hypothetical replications of TOEFL scores comparisons. When 95% CIs are drawn around each d ′ value (shown as bars in Figure 1 ), 95% of them include the parameter δ (horizontal line). However, many fewer than 95% of the d ′ values fall within the first CI (projected as broken lines). This is because there are two sources of sampling error: the error realized in the obtained measure and all the future errors in the hypothetical replications. It requires considerable conceptual effort to avoid this misconception when using CIs. A discussion of CIs and their proper interpretation may be found both in Estes (1997) and Thompson (2002) , and in the excellent reviews by Cumming and associates—in this issue, in other journals, (e.g., Cumming & Finch, 2001 ) and on his Web site ( http://www.latrobe.edu.au/psy/staff/cumming.html ).

An external file that holds a picture, illustration, etc.
Object name is nihms79997f1.jpg

Hypothetical results of 100 replications of a comparison between two group means. Each data point was sampled from a normal distribution, its mean ( δ ) represented by the solid horizontal line. Confidence intervals around each data point were calculated using the standard deviation of the data. The dashed horizontal lines are projections of a confidence interval centered on the first data point.

Problem 2: Power

What if d ′ > 0, but p = .05 or .07 or even .25? Should the old method be preserved because there was no real difference? Recall that p indicates the probability that a difference ≥ d ′ would be obtained by chance, which is not the same as the expected magnitude of that difference. A high p value does not necessarily mean that the new method is not an improvement over the old; rather, there exists the possibility that the statistical test was not powerful enough to render a real difference significant. The conventional solution to this problem is to choose the size of each treatment group based on the results of a power analysis conducted prior to the comparison of teaching methods. The inputs to a power analysis are the hypostasized population effect size ( δ *) and the probability α of rejecting the null given that d ′ = δ *. In a choice situation, δ * could take on the value of the minimum effect size that would justify adoption of the more costly alternative; if no difference is detected, the less costly alternative would be chosen, decision makers knowing that there is a probability β of having made the wrong choice (Type II error). Thus, two criteria must be established: a minimum d ′ value that would justify the adoption of the new method and the probability, or willingness, of missing a significant effect ( β ≈ 20% is conventional). As an example, suppose that the power analysis says that at least 1000 students should be recruited for each ESL teaching method group. If these group sizes are not physically or financially possible, the probability of making a Type II error would increase. But even if it were possible to recruit 1000 students for each group, we may be back close to where we started, obtaining an “insignificant” p value of .07 regardless! Thus, conducting a power analysis does not guarantee that NHST will guide us to a resolute decision.

An Alternative to NHST: Probability of Replication

The NHST routine provides a probability ( p ) associated with a statistic ( d ′), given a null hypothesis ( H 0 ). When factoring in decision-relevant variables such as the difficulty of switching teaching methods, the statistics d ′ and p , by themselves, are inadequate. Confidence intervals appear to show the probability of a minimum and maximum mean difference in future comparisons, but they do not. Even with power analysis, marginal significance is always possible, confounding any decision. To solve these problems, we suggest the adoption of a statistic that provides what confidence intervals do not and abandons the discontinuous acceptance criteria on which the issue of marginal significance hinges. That statistic is the estimated probability of replication ( p rep ). While in this article we cover the basic rationale of p rep , further reading can provide a tutorial ( Killeen, 2005c , in press ), a more detailed discussion ( Killeen, 2005a , 2005b ), and an alternate decision-theoretic approach ( Killeen, 2006 ).

Consider the information provided by the p value in a NHST. The null hypothesis ( H 0 ) is represented in Figure 2A by a distribution of effects centered on zero; the shaded area represents the probability of sampling an effect d 2 ′ larger than the one obtained ( d 1 ′ ) if the null hypothesis were true, that is, p ( d 2 ′ > d 1 ′ ∣ H 0 ) . This is the conventional “level of significance.” A particular level of significance connotes the likelihood of the null hypothesis, p ( H 0 ∣ d 1 ′ ) , by pointing at the un likelihood of an obtained effect. Note, however, that level of significance p and likelihood of H 0 are not the same; even if they were, the probability of effects sampled from H 0 in future comparisons does not translate into an expectation of the size of future effects on which to base a decision. A decision would be better informed by knowing the probability of obtaining another positive effect in replication, one that exceeds some minimum effect size d s ′ : P R = p ( d ′ > d s ′ ∣ δ ) , where d ′ is any effect size greater than the minimum effect size that would support a positive decision (e.g., adopt new method), d s ′ , and δ is the true mean of the distribution of the effects. In the example of ESL teaching methods, this is the probability that a minimum acceptable improvement in ESL performance would be obtained, if the new method were adopted. If this probability is satisfactory, the new method should be adopted; otherwise, it should be rejected.

An external file that holds a picture, illustration, etc.
Object name is nihms79997f2.jpg

The shaded areas in these panels show the following: (A) probability ( p ) of obtaining an effect larger than d 1 ′ under a null distribution. (B) Probability of obtaining an effect larger than zero (the probability of replication, or PR ) under the true normal distribution of effects with mean δ ; d 1 ′ is a sample from this distribution, with sampling error Δ 1 . (C) Estimate of PR ( p rep ) under the distribution of effects estimated from d 1 ′ . (D) Estimate of the probability of a replication having an effect size greater than criterion d s ′ .

Figure 2B illustrates this approach graphically, where the true distribution of effects is represented as a normal distribution ( Hedges & Olkin, 1985 ), centered on the true (but unknown) population effect size δ ; sampling error for a measurement d 1 ′ is represented by

For the moment, the minimum difference criterion, d s ′ , has been set to zero for simplicity (i.e., any improvement supports a positive decision). The shaded area to the right of d s ′ = 0 in Figure 2B represents PR . Note that (a) in the calculation of PR , the null hypothesis H 0 may be disregarded, but (b) without δ , PR cannot be calculated. Because δ is unknown, we must estimate PR based solely on d 1 ′ .

Effect sizes are approximately normally distributed, with a standard error approximately equal to the square root of

where n 1 and n 2 are the sample sizes of each treatment, n + n 1 + n 2 , and − 1 < d 1 ′ < 1 ( Hedges & Olkin, 1985 ; Killeen, 2005a ). When n 1 = n 2 , this further simplifies to

With these estimates of effect size ( d 1 ′ ) and its variance ( σ d ′ 2 ), an estimate of PR may be calculated. We call this estimate p rep .

Calculating p rep

The path leading to this statistic is somewhat technical ( Appendix A ), but an intuitive understanding may be gained from Figure 2 . The variance of the sampling distribution for the original effect size is σ d ′ 2 . This error is incurred twice: once in the original estimate (shown as the sampling error Δ 1 ) and again as the sampling error of the replication Δ 2 . The expected value of the squares of these errors corresponds to the variance of the sampling distributions. Because they are incurred twice, these variances summate. Therefore, the variance of the replication distribution, shown in the bottom of Figure 2 , is

and the probability distribution for replications is

The estimated probability of a replication, p ( d 2 ′ > 0 ∣ d 1 ′ ) or p rep , is the area of the distribution for which d 1 ′ is greater than 0, shaded in Figure 2C . This is equivalent to the cumulative probability in a normal distribution up to

In a spreadsheet program such as Microsoft Excel, simply input the obtained z value in a NORMS-DIST() function to obtain p rep . Appendix B summarizes the steps to obtain p rep from group means and standard deviations.

Let us illustrate the calculation of p rep with a hypothetical example. Suppose that the two methods of teaching ESL were tested in two groups of 127 students ( n OLD = n NEW = 127; n = 254). Both groups are matched on all relevant variables (e.g., baseline TOEFL scores, age, gender, educational background, etc.). After a semester of exposure to their group’s teaching method, each student is tested using CBT-TOEFL, which has a range of possible scores between 40 (worst) and 300 (best). The mean TOEFL score obtained by Group OLD was 206.1, whereas the mean score of Group NEW was 240. Pooled standard deviation s POOLED = 169.3; thus

These results indicate that, on average, almost nine out of every ten groups of 127 students may be expected to obtain higher average TOEFL scores when exposed to the new method rather than the old.

Incorporating a Minimum Difference Criterion ( d s ′ )

So far we have considered the situation where the minimum difference criterion d s ′ = 0 , that is, where any positive difference in scores justifies the adoption of the more costly alternative. It is more realistic, however, to work under the assumption that very small positive differences, however certain, may not justify the cost of the better option. In that case, the first step is to calculate the difference in costs between the two alternatives and define the minimum effect size d s ′ that would justify the cost differential. The function that relates cost differentials and minimum difference criteria could take any shape; it may be continuous, as in the case where performance is evaluated by the average TOEFL score obtained, or it may be discontinuous, as in the case where foreign teaching assistants must obtain a minimum TOEFL score to become certified. For illustration purposes and simplicity, let us assume the latter. Suppose that the old ESL teaching method yielded an average TOEFL score of 206.1, but students need at least a 223 to qualify for teaching assistantships. This would represent a minimum difference between passing and failing of 16.9 points. For generality’s sake, we specify the minimum difference as an effect size, which is the difference divided by the pooled standard deviation ( s POOLED ). An estimate of s POOLED , in this case, can be obtained from prior TOEFLs. Using the values from the examples above ( M NEW = 240, M OLD = 206.1, s POOLED = 169.3), if the required minimum improvement in TOEFL scores is 16.9 points, and, as in the above example, M NEW − M OLD = 33.9, then d s ′ = 16.9 / 169.3 = 0.1. Thus, 0.1 is the minimum effect required from the new method.

Just as p rep is an estimate of the probability of obtaining a second effect greater than zero, p support is an estimate of the probability of obtaining a second effect greater than d s ′ . This probability is the area of the distribution for which d ′ is greater than d s ′ , shaded in Figure 2D and equivalent to the cumulative probability in a normal distribution up to

This z value may be input into a table of normal deviates, or a spreadsheet, to obtain p support . From Equation 8 :

This means that we expect about seven of every ten groups of 127 students to increase their average TOEFL score by at least 16.9 points, and therefore qualify for a teaching assistantship, when exposed to the new method rather than the old.

From p to p rep and p support : Two Examples from Psychology in the Schools

In a study of mathematics proficiency and frustration response, Scime and Norvilitis (2006) asked 64 children with and without attention deficit hyperactivity disorder (ADHD) to complete a complex puzzle task and arithmetic problems of increasing difficulty. They compared the ADHD and non-ADHD groups across 17 ratings of task performance, reaction to frustration, emotional competence, and proficiency in mathematics. Individual t tests established the significance of between-group differences on each rating category. Because of the large number of t tests, the researchers applied a Bonferroni correction to minimize Type I errors, elevating the significance criterion to α = .003. Given the high criterion level, only three significant differences were detected.

Even when informing an intervention decision is not strictly the intent of a comparison, remaining agnostic about nonsignificant differences may prove difficult. In comparing the mathematics proficiency of ADHD and non-ADHD children, Scime and Norvilitis (2006) found a significant difference in overall completion rates, but not in overall accuracy. Based on these results, Scime and Norvilitis concluded that “children with ADHD did not complete as many items but were equally accurate on those that they did complete” (p. 383, italics added). This conclusion, however, is not supported by the data, as shown in Table 1 : Children without ADHD were more accurate in their problem solving than those with ADHD, although the difference was not statistically significant ( p > .003). In fact, p rep for this comparison suggests that in 65% of similar comparisons, the children without ADHD would outperform the children with ADHD. This percentage is, undoubtedly, much smaller than the proportion of tests in which ADHD is predicted to be associated with lower completion rates (99%). However, the fact that accuracy is more similar across groups than completion rates does not translate into equal performance. Whether 65% is a meaningful failure rate or not is a theoretical and practical consideration, not a statistical one.

Statistical Analysis of Selected Tests From Two Studies

StudyVariable ( ) ( )
Mathematics-Accuracy6483.81 (15.44)86.44 (18.71)0.15.285.649
Mathematics-Completion6426.88 (22.14)44.52 (15.69)0.98.002.994
TMMS-C Emotional Attention643.30 (0.43)3.65 (0.56)0.67.028.958
TMMS-C Emotional Clarity643.63 (0.69)3.85 (0.64)0.34.085.806
Reading (UMSP)942.63 (0.91)2.81 (0.79)0.21.163.753
Reading (Comparison)1022.54 (1.02)2.73 (0.89)0.20.169.748
Math (UMSP)942.41 (0.98)2.75 (0.85)0.36.044.884
Math (Comparison)1022.78 (1.02)2.67 (0.95)−0.11n.a..353

Note . Scime and Norvilitis (2006) : Group 1 = ADHD, Group 2 = No-ADHD; Mathematics scores were based on total number of problems completed and number of problems completed correctly; TMMS-C = Trait Meta-Mood Scale for Children. Linares et al. (2005) : Group 1 = Grades at baseline, Group 2 = Grades at Year 2; reading and math grades are on a range of 1 ( unsatisfactory ) to 4 ( excellent ). All p values were estimated from published data using one-tailed t tests for independent samples.

The high significance criterion imposed by Scime and Norvilitis (2006) did yield some differences marginally significant. For example, in the Emotional Attention variable of the Trait Meta-Mood Scale for Children (TMMS-C), we may expect children without ADHD to score higher than children with ADHD in 96% of similar tests. Despite such a high percentage, Scime and Norvilitis report the difference as not significant. Contradictions such as this one may be attributed to the stringent familywise error correction established by the researchers, although other seemingly replicable results would have been reported as not significant even under more lenient criteria (e.g., Emotional Clarity: p = .085; p rep = 81%). Like p , p rep is not immune to familywise errors. As the number of comparisons increase, so too does the likelihood that an extreme p rep value will be obtained due to sampling error; yet the best estimate of any given PR is its corresponding p rep . When retesting is not possible, any decision based on multiple comparisons should be tempered by these considerations.

In a treatment comparison study, Linares et al. (2005) analyzed 14 outcome measures in fourth-graders, comparing students in a school that adopted the Unique Minds School Program (UMSP) with one that did not, over a 2-year period. Two of the outcome measures were academic grades in reading and in mathematics. Linares et al. reported no significant differences in reading grades, but a Time × School interaction in mathematics. This interaction suggests that students in the UMSP school showed larger improvement in mathematics grades than students in the comparison school (see Table 1 ). The p rep statistic allows us to look at this interaction in a way that informs the UMSP adoption decision. A comparison of students’ grades at baseline versus Year 2 indicates that, over the course of 2 years, we can expect 75% and 88% of fourth-graders exposed to UMSP to improve their reading and math grades, respectively. To control for student maturation, we compare these improvement percentages with those expected from a non-UMSP school: Results indicate that 75% of students improve in reading and only 35% in math. Subtracting the expected improvement percentage in non-UMSP from UMSP shows that slightly more than half of the students would obtain better math grades after 2 years if exposed to the UMSP; no effect is expected in reading grades.

Despite the strong effects obtained, the difference in baseline grades across schools undermines an unequivocal interpretation of the Time × School interaction obtained by Linares et al. (2005) . The implementation of the UMSP is confounded with undetermined variables responsible for baseline differences. Rescaling the grades to equalize both groups at baseline may solve this problem. Another way to assess Linares et al.’s results is by determining the expected percentage of UMSP students that will increase their math and reading grades at least to the level of the comparison school in their second year (sixth grade). The minimum grade improvements (0.10 in reading, 0.26 in mathematics) were divided by the corresponding s POOLED for UMSP grades to obtain d s ′ for reading and math ( Table 2 ). The obtained values of p support suggest that three out of every five groups of UMSP students will obtain higher grades in both math and reading in the sixth grade than non-UMSP sixth-graders.

Computation of p support for Two Tests in Linares et al. (2005)

StudyVariable
Reading (UMSP)0.210.12.619
Math (UMSP)0.360.28.611

Note . Values of d s ′ were based on mean performance of comparison school at Year 2. See text for explanation.

The Loss of False Certainty Is a Gain

Something seems to be lost in using p rep and p support in place of p . Even though nonsignificant differences may leave us in a limbo of indecision, significant differences appear to inform researchers about some true effect, embodied in the low probability that such differences could be obtained by chance. Who would doubt the reality of an effect when, for instance, p < .001? This false sense of certainty is derived from the rather unlikely assumption that, for any comparison, there is only one real and detectable effect, instead of a distribution of obtainable effects with a hypothetical mean and dispersion. Only the assumption of an underlying binary status of effects (real vs. not real) would support a routine that allows for only two possible outcomes: Either the reality of a tested effect is detected (i.e., it is “significant”) and we can safely assume its existence or it is not detected (i.e., it is “not significant”) and nothing can be said about its existence. Under NHST, researchers are asked to operate as if an effect was a single value of which we may only know its sign. It is for this reason that Fisher (1959) suggested null hypothesis testing as a method to detect unlikely events that deserve further examination —not as a substitute for that examination.

The difference between two delimited populations (e.g., senior high school students in the state of Louisiana vs. Texas in 2004) over an attribute (e.g., mean grade point average) may be reasonably represented as a single value. This is, however, a rare case. Generally, researchers are interested in differences between populations that may change from test to test. In our example of ESL teaching methods, a retest would certainly involve different students that may respond differently to each teaching method. Which of the two tests would yield results closer to the real difference between teaching methods? This would be a moot question if effects were thought of as normally distributed real differences, rather than estimates of a single real difference, that is, if we admit that a teaching method may be beneficial most of the time but not necessarily all of the time. The uncertainty inherent in a probability distribution may undermine the resoluteness that is expected to guide practical action; however, resolute and statistically sophisticated decisions based on false assumptions are but esoteric routes to failure. We argued for and described a simple way to reduce such discomfiture by estimating the probability distribution of replication, based on d 1 ′ and σ R . The closure provided by a p value is misleading. In contrast, p rep provides the basis for a cautious, fully informed decision, one open to graded assent and recalibration against minimal standards such as p support . Most importantly, perhaps, p rep provides a clearer level of communication to all participants regarding the complicated decisions that the educational community faces today.

Acknowledgments

This research was supported by NIMH grant # 1R01MH066860 and NSF grant IBN 0236821.

Appendix A: Justification

To calculate the probability of a successful replication, we must be able to calculate the probability of any particular value of a replicate effect d 2 ′ , given the population mean effect δ . But δ is unknown. The probability of δ having any particular value can only be estimated from prior information and from current information such as d 1 ′ . Attempts to leapfrog over the nuisance parameter δ and directly determine the probability that d 2 ′ will take any particular value d 2 ∗ given d 1 ′ have not clearly succeeded ( Macdonald, 2005 ; Seidenfeld, 1979 ), unless they invoke Bayesian updating of prior information. This is because the value of d 2 ′ is not independent of the value of d 1 ′ . So instead we must go by the indirect route of estimating p ( d 2 ′ ∣ d 1 ′ ) in terms of p ( d 2 ′ ∣ δ ) and p ( δ ∣ d 1 ′ ) . The latter is derived from p ( d 1 ′ ∣ δ ) and p ( δ ) using Bayes Theorem. The problem with such updating is that (a) it commits us to agree that parameters can themselves have distributions and (b) it requires knowledge of p ( δ )—and in determining that we eventually regress to conditions about which we are ignorant. Justifying how to formulate that ignorance has been the bane of progress. Unlike frequentists, Bayesians are willing to bite both these bullets. One way of engaging “ignorance” priors is with an appropriate ( conjugate ) distribution with an arbitrarily large variance, which entails that any prior value for a parameter such as δ is equally likely ( Doros & Geier, 2005 ; O’Hagan & Forster, 2004 , pp. 89–91).

The advantage of this approach is that it can naturally take advantage of available prior information, enhancing the accuracy of our predictions. Alternatively, as argued here and by Maurer (2004 , p. 17), the engagement of informative priors “can obfuscate formal tests [of the data under inspection] by including information not specifically contained within the experiment itself”; for primary credentialing of results, ignorance priors and the calculations of p rep given in text and below, are ideal.

Appendix B: Calculation

To calculate p rep for the difference of two group means:

  • Divide the difference of the two group means by the pooled standard deviation to compute the effect size d 1 ′ = ( M 1 − M 2 ) / s POOLED .

where n i and s i are the number of subjects and the standard deviation of group i , respectively.

  • Using Microsoft Excel, input in a cell the command NORMSDIST ( d 1 ′ / σ R ). This is equivalent to consulting a normal probability table for the cumulative probability up to z = d 1 ′ / σ R .

If the data are from regression analyses, the standard t value is ( Rosenthal, 1994 ; Rosnow & Rosenthal, 2003 )

this may be converted to p rep by evaluating 1 − F ( t 2 ) , where F ( x ) returns the tail probability of the t distribution. In a spreadsheet such as Excel this can be computed with p rep = 1 − TDIST ( t / 2 , df , 1 ) . Other useful conversions are d ′= 2 r (1 − r 2 ) −1/2 ( Rosenthal, 1994 ), and d ′= t [1/ n 1 + 1/ n 2 ] 1/2 for the simple two independent group case, and d ′= t r [(1 − r )/ n 1 (1 − r )/ n 2 ] 1/2 for a repeated measures t , where r is the correlation between the measures ( Cortina & Nouri, 2000 ).

  • Cortina JM, Nouri H. Effect size for ANOVA designs. Thousand Oaks, CA: Sage; 2000. [ Google Scholar ]
  • Cumming G, Finch S. A primer on the understanding, use and calculation of confidence intervals based on central and noncentral distributions. Educational and Psychological Measurement. 2001; 61 :532–575. [ Google Scholar ]
  • Doros G, Geier AB. Comment on “An alternative to null hypothesis significance tests. Psychological Science. 2005; 16 :1005–1006. [ PubMed ] [ Google Scholar ]
  • Estes WK. On the communication of information by displays of standard errors and confidence intervals. Psy-chonomic Bulletin & Review. 1997; 4 :330–341. [ Google Scholar ]
  • Fisher RA. Statistical methods and scientific inference. New York: Hafner; 1959. [ Google Scholar ]
  • Hedges LV, Olkin I. Statistical methods for meta-analysis. New York: Academic Press; 1985. [ Google Scholar ]
  • Killeen PR. An alternative to null hypothesis significance tests. Psychological Science. 2005a; 16 :345–353. [ PMC free article ] [ PubMed ] [ Google Scholar ]
  • Killeen PR. Replicability, confidence, and priors. Psychological Science. 2005b; 16 :1009–1012. [ PMC free article ] [ PubMed ] [ Google Scholar ]
  • Killeen PR. Tea-tests. The General Psychologist. 2005c; 40 :16–19. [ Google Scholar ]
  • Killeen PR. Beyond statistical inference: A decision theory for science. Psychonomic Bulletin & Review. 2006; 13 :549–569. [ PMC free article ] [ PubMed ] [ Google Scholar ]
  • Killeen PR. The probability of replication: Its logic, justification, and calculation. In: Osborne JW, editor. Best practices in quantitative methods. Thousand Oaks, CA: Sage; in press. [ Google Scholar ]
  • Linares LO, Rosbruch N, Stern MB, Edwards ME, Walker G, Abikoff HB, et al. Developing cognitive-social-emotional competencies to enhance academic learning. Psychology in the Schools. 2005; 42 :405– 417. [ Google Scholar ]
  • Macdonald RR. Why replication probabilities depend on prior probability distributions. A rejoinder to Killeen (2005) Psychological Science. 2005; 16 :1007–1008. [ PubMed ] [ Google Scholar ]
  • Maurer BA. Models of scientific inquiry and statistical practice: Implications for the structure of scientific knowledge. In: Taper ML, Lele SR, editors. The nature of scientific evidence: Statistical, philosophical, and empirical considerations. Chicago: University of Chicago Press; 2004. pp. 17–50. [ Google Scholar ]
  • O’Hagan A, Forster J. Bayesian inference. 2B. Vol. 2. New York: Oxford University Press; 2004. Kendall’s advanced theory of statistics. [ Google Scholar ]
  • Rosenthal R. Parametric measures of effect size. In: Cooper H, Hedges LV, editors. The handbook of research synthesis. New York: Russell Sage Foundation; 1994. pp. 231–244. [ Google Scholar ]
  • Rosnow RL, Rosenthal R. Effect sizes for experimenting psychologists. Canadian Journal of Experimental Psychology. 2003; 57 :221–237. [ PubMed ] [ Google Scholar ]
  • Royall R. Statistical evidence: A likelihood paradigm. London: Chapman & Hall; 1997. [ Google Scholar ]
  • Scime M, Norvilitis JM. Task performance and response to frustration in children with Attention Deficit Hyperactivity Disorder. Psychology in the Schools. 2006; 43 :377–386. [ Google Scholar ]
  • Seidenfeld T. Philosophical problems of statistical inference: Learning from R.A. Fisher. London: D. Reidel; 1979. [ Google Scholar ]
  • Thompson B. What future quantitative social science research could look like: Confidence intervals for effect sizes. Educational Researchers. 2002; 31 :25–32. [ Google Scholar ]

Have a language expert improve your writing

Run a free plagiarism check in 10 minutes, generate accurate citations for free.

  • Knowledge Base
  • Null and Alternative Hypotheses | Definitions & Examples

Null & Alternative Hypotheses | Definitions, Templates & Examples

Published on May 6, 2022 by Shaun Turney . Revised on June 22, 2023.

The null and alternative hypotheses are two competing claims that researchers weigh evidence for and against using a statistical test :

  • Null hypothesis ( H 0 ): There’s no effect in the population .
  • Alternative hypothesis ( H a or H 1 ) : There’s an effect in the population.

Table of contents

Answering your research question with hypotheses, what is a null hypothesis, what is an alternative hypothesis, similarities and differences between null and alternative hypotheses, how to write null and alternative hypotheses, other interesting articles, frequently asked questions.

The null and alternative hypotheses offer competing answers to your research question . When the research question asks “Does the independent variable affect the dependent variable?”:

  • The null hypothesis ( H 0 ) answers “No, there’s no effect in the population.”
  • The alternative hypothesis ( H a ) answers “Yes, there is an effect in the population.”

The null and alternative are always claims about the population. That’s because the goal of hypothesis testing is to make inferences about a population based on a sample . Often, we infer whether there’s an effect in the population by looking at differences between groups or relationships between variables in the sample. It’s critical for your research to write strong hypotheses .

You can use a statistical test to decide whether the evidence favors the null or alternative hypothesis. Each type of statistical test comes with a specific way of phrasing the null and alternative hypothesis. However, the hypotheses can also be phrased in a general way that applies to any test.

Here's why students love Scribbr's proofreading services

Discover proofreading & editing

The null hypothesis is the claim that there’s no effect in the population.

If the sample provides enough evidence against the claim that there’s no effect in the population ( p ≤ α), then we can reject the null hypothesis . Otherwise, we fail to reject the null hypothesis.

Although “fail to reject” may sound awkward, it’s the only wording that statisticians accept . Be careful not to say you “prove” or “accept” the null hypothesis.

Null hypotheses often include phrases such as “no effect,” “no difference,” or “no relationship.” When written in mathematical terms, they always include an equality (usually =, but sometimes ≥ or ≤).

You can never know with complete certainty whether there is an effect in the population. Some percentage of the time, your inference about the population will be incorrect. When you incorrectly reject the null hypothesis, it’s called a type I error . When you incorrectly fail to reject it, it’s a type II error.

Examples of null hypotheses

The table below gives examples of research questions and null hypotheses. There’s always more than one way to answer a research question, but these null hypotheses can help you get started.

( )
Does tooth flossing affect the number of cavities? Tooth flossing has on the number of cavities. test:

The mean number of cavities per person does not differ between the flossing group (µ ) and the non-flossing group (µ ) in the population; µ = µ .

Does the amount of text highlighted in the textbook affect exam scores? The amount of text highlighted in the textbook has on exam scores. :

There is no relationship between the amount of text highlighted and exam scores in the population; β = 0.

Does daily meditation decrease the incidence of depression? Daily meditation the incidence of depression.* test:

The proportion of people with depression in the daily-meditation group ( ) is greater than or equal to the no-meditation group ( ) in the population; ≥ .

*Note that some researchers prefer to always write the null hypothesis in terms of “no effect” and “=”. It would be fine to say that daily meditation has no effect on the incidence of depression and p 1 = p 2 .

The alternative hypothesis ( H a ) is the other answer to your research question . It claims that there’s an effect in the population.

Often, your alternative hypothesis is the same as your research hypothesis. In other words, it’s the claim that you expect or hope will be true.

The alternative hypothesis is the complement to the null hypothesis. Null and alternative hypotheses are exhaustive, meaning that together they cover every possible outcome. They are also mutually exclusive, meaning that only one can be true at a time.

Alternative hypotheses often include phrases such as “an effect,” “a difference,” or “a relationship.” When alternative hypotheses are written in mathematical terms, they always include an inequality (usually ≠, but sometimes < or >). As with null hypotheses, there are many acceptable ways to phrase an alternative hypothesis.

Examples of alternative hypotheses

The table below gives examples of research questions and alternative hypotheses to help you get started with formulating your own.

Does tooth flossing affect the number of cavities? Tooth flossing has an on the number of cavities. test:

The mean number of cavities per person differs between the flossing group (µ ) and the non-flossing group (µ ) in the population; µ ≠ µ .

Does the amount of text highlighted in a textbook affect exam scores? The amount of text highlighted in the textbook has an on exam scores. :

There is a relationship between the amount of text highlighted and exam scores in the population; β ≠ 0.

Does daily meditation decrease the incidence of depression? Daily meditation the incidence of depression. test:

The proportion of people with depression in the daily-meditation group ( ) is less than the no-meditation group ( ) in the population; < .

Null and alternative hypotheses are similar in some ways:

  • They’re both answers to the research question.
  • They both make claims about the population.
  • They’re both evaluated by statistical tests.

However, there are important differences between the two types of hypotheses, summarized in the following table.

A claim that there is in the population. A claim that there is in the population.

Equality symbol (=, ≥, or ≤) Inequality symbol (≠, <, or >)
Rejected Supported
Failed to reject Not supported

Receive feedback on language, structure, and formatting

Professional editors proofread and edit your paper by focusing on:

  • Academic style
  • Vague sentences
  • Style consistency

See an example

is rejecting null hypothesis good

To help you write your hypotheses, you can use the template sentences below. If you know which statistical test you’re going to use, you can use the test-specific template sentences. Otherwise, you can use the general template sentences.

General template sentences

The only thing you need to know to use these general template sentences are your dependent and independent variables. To write your research question, null hypothesis, and alternative hypothesis, fill in the following sentences with your variables:

Does independent variable affect dependent variable ?

  • Null hypothesis ( H 0 ): Independent variable does not affect dependent variable.
  • Alternative hypothesis ( H a ): Independent variable affects dependent variable.

Test-specific template sentences

Once you know the statistical test you’ll be using, you can write your hypotheses in a more precise and mathematical way specific to the test you chose. The table below provides template sentences for common statistical tests.

( )
test 

with two groups

The mean dependent variable does not differ between group 1 (µ ) and group 2 (µ ) in the population; µ = µ . The mean dependent variable differs between group 1 (µ ) and group 2 (µ ) in the population; µ ≠ µ .
with three groups The mean dependent variable does not differ between group 1 (µ ), group 2 (µ ), and group 3 (µ ) in the population; µ = µ = µ . The mean dependent variable of group 1 (µ ), group 2 (µ ), and group 3 (µ ) are not all equal in the population.
There is no correlation between independent variable and dependent variable in the population; ρ = 0. There is a correlation between independent variable and dependent variable in the population; ρ ≠ 0.
There is no relationship between independent variable and dependent variable in the population; β = 0. There is a relationship between independent variable and dependent variable in the population; β ≠ 0.
Two-proportions test The dependent variable expressed as a proportion does not differ between group 1 ( ) and group 2 ( ) in the population; = . The dependent variable expressed as a proportion differs between group 1 ( ) and group 2 ( ) in the population; ≠ .

Note: The template sentences above assume that you’re performing one-tailed tests . One-tailed tests are appropriate for most studies.

If you want to know more about statistics , methodology , or research bias , make sure to check out some of our other articles with explanations and examples.

  • Normal distribution
  • Descriptive statistics
  • Measures of central tendency
  • Correlation coefficient

Methodology

  • Cluster sampling
  • Stratified sampling
  • Types of interviews
  • Cohort study
  • Thematic analysis

Research bias

  • Implicit bias
  • Cognitive bias
  • Survivorship bias
  • Availability heuristic
  • Nonresponse bias
  • Regression to the mean

Hypothesis testing is a formal procedure for investigating our ideas about the world using statistics. It is used by scientists to test specific predictions, called hypotheses , by calculating how likely it is that a pattern or relationship between variables could have arisen by chance.

Null and alternative hypotheses are used in statistical hypothesis testing . The null hypothesis of a test always predicts no effect or no relationship between variables, while the alternative hypothesis states your research prediction of an effect or relationship.

The null hypothesis is often abbreviated as H 0 . When the null hypothesis is written using mathematical symbols, it always includes an equality symbol (usually =, but sometimes ≥ or ≤).

The alternative hypothesis is often abbreviated as H a or H 1 . When the alternative hypothesis is written using mathematical symbols, it always includes an inequality symbol (usually ≠, but sometimes < or >).

A research hypothesis is your proposed answer to your research question. The research hypothesis usually includes an explanation (“ x affects y because …”).

A statistical hypothesis, on the other hand, is a mathematical statement about a population parameter. Statistical hypotheses always come in pairs: the null and alternative hypotheses . In a well-designed study , the statistical hypotheses correspond logically to the research hypothesis.

Cite this Scribbr article

If you want to cite this source, you can copy and paste the citation or click the “Cite this Scribbr article” button to automatically add the citation to our free Citation Generator.

Turney, S. (2023, June 22). Null & Alternative Hypotheses | Definitions, Templates & Examples. Scribbr. Retrieved September 23, 2024, from https://www.scribbr.com/statistics/null-and-alternative-hypotheses/

Is this article helpful?

Shaun Turney

Shaun Turney

Other students also liked, inferential statistics | an easy introduction & examples, hypothesis testing | a step-by-step guide with easy examples, type i & type ii errors | differences, examples, visualizations, what is your plagiarism score.

Logo for BCcampus Open Publishing

Want to create or adapt books like this? Learn more about how Pressbooks supports open publishing practices.

Chapter 13: Inferential Statistics

Understanding Null Hypothesis Testing

Learning Objectives

  • Explain the purpose of null hypothesis testing, including the role of sampling error.
  • Describe the basic logic of null hypothesis testing.
  • Describe the role of relationship strength and sample size in determining statistical significance and make reasonable judgments about statistical significance based on these two factors.

The Purpose of Null Hypothesis Testing

As we have seen, psychological research typically involves measuring one or more variables for a sample and computing descriptive statistics for that sample. In general, however, the researcher’s goal is not to draw conclusions about that sample but to draw conclusions about the population that the sample was selected from. Thus researchers must use sample statistics to draw conclusions about the corresponding values in the population. These corresponding values in the population are called  parameters . Imagine, for example, that a researcher measures the number of depressive symptoms exhibited by each of 50 clinically depressed adults and computes the mean number of symptoms. The researcher probably wants to use this sample statistic (the mean number of symptoms for the sample) to draw conclusions about the corresponding population parameter (the mean number of symptoms for clinically depressed adults).

Unfortunately, sample statistics are not perfect estimates of their corresponding population parameters. This is because there is a certain amount of random variability in any statistic from sample to sample. The mean number of depressive symptoms might be 8.73 in one sample of clinically depressed adults, 6.45 in a second sample, and 9.44 in a third—even though these samples are selected randomly from the same population. Similarly, the correlation (Pearson’s  r ) between two variables might be +.24 in one sample, −.04 in a second sample, and +.15 in a third—again, even though these samples are selected randomly from the same population. This random variability in a statistic from sample to sample is called  sampling error . (Note that the term error  here refers to random variability and does not imply that anyone has made a mistake. No one “commits a sampling error.”)

One implication of this is that when there is a statistical relationship in a sample, it is not always clear that there is a statistical relationship in the population. A small difference between two group means in a sample might indicate that there is a small difference between the two group means in the population. But it could also be that there is no difference between the means in the population and that the difference in the sample is just a matter of sampling error. Similarly, a Pearson’s  r  value of −.29 in a sample might mean that there is a negative relationship in the population. But it could also be that there is no relationship in the population and that the relationship in the sample is just a matter of sampling error.

In fact, any statistical relationship in a sample can be interpreted in two ways:

  • There is a relationship in the population, and the relationship in the sample reflects this.
  • There is no relationship in the population, and the relationship in the sample reflects only sampling error.

The purpose of null hypothesis testing is simply to help researchers decide between these two interpretations.

The Logic of Null Hypothesis Testing

Null hypothesis testing  is a formal approach to deciding between two interpretations of a statistical relationship in a sample. One interpretation is called the   null hypothesis  (often symbolized  H 0  and read as “H-naught”). This is the idea that there is no relationship in the population and that the relationship in the sample reflects only sampling error. Informally, the null hypothesis is that the sample relationship “occurred by chance.” The other interpretation is called the  alternative hypothesis  (often symbolized as  H 1 ). This is the idea that there is a relationship in the population and that the relationship in the sample reflects this relationship in the population.

Again, every statistical relationship in a sample can be interpreted in either of these two ways: It might have occurred by chance, or it might reflect a relationship in the population. So researchers need a way to decide between them. Although there are many specific null hypothesis testing techniques, they are all based on the same general logic. The steps are as follows:

  • Assume for the moment that the null hypothesis is true. There is no relationship between the variables in the population.
  • Determine how likely the sample relationship would be if the null hypothesis were true.
  • If the sample relationship would be extremely unlikely, then reject the null hypothesis  in favour of the alternative hypothesis. If it would not be extremely unlikely, then  retain the null hypothesis .

Following this logic, we can begin to understand why Mehl and his colleagues concluded that there is no difference in talkativeness between women and men in the population. In essence, they asked the following question: “If there were no difference in the population, how likely is it that we would find a small difference of  d  = 0.06 in our sample?” Their answer to this question was that this sample relationship would be fairly likely if the null hypothesis were true. Therefore, they retained the null hypothesis—concluding that there is no evidence of a sex difference in the population. We can also see why Kanner and his colleagues concluded that there is a correlation between hassles and symptoms in the population. They asked, “If the null hypothesis were true, how likely is it that we would find a strong correlation of +.60 in our sample?” Their answer to this question was that this sample relationship would be fairly unlikely if the null hypothesis were true. Therefore, they rejected the null hypothesis in favour of the alternative hypothesis—concluding that there is a positive correlation between these variables in the population.

A crucial step in null hypothesis testing is finding the likelihood of the sample result if the null hypothesis were true. This probability is called the  p value . A low  p  value means that the sample result would be unlikely if the null hypothesis were true and leads to the rejection of the null hypothesis. A high  p  value means that the sample result would be likely if the null hypothesis were true and leads to the retention of the null hypothesis. But how low must the  p  value be before the sample result is considered unlikely enough to reject the null hypothesis? In null hypothesis testing, this criterion is called  α (alpha)  and is almost always set to .05. If there is less than a 5% chance of a result as extreme as the sample result if the null hypothesis were true, then the null hypothesis is rejected. When this happens, the result is said to be  statistically significant . If there is greater than a 5% chance of a result as extreme as the sample result when the null hypothesis is true, then the null hypothesis is retained. This does not necessarily mean that the researcher accepts the null hypothesis as true—only that there is not currently enough evidence to conclude that it is true. Researchers often use the expression “fail to reject the null hypothesis” rather than “retain the null hypothesis,” but they never use the expression “accept the null hypothesis.”

The Misunderstood  p  Value

The  p  value is one of the most misunderstood quantities in psychological research (Cohen, 1994) [1] . Even professional researchers misinterpret it, and it is not unusual for such misinterpretations to appear in statistics textbooks!

The most common misinterpretation is that the  p  value is the probability that the null hypothesis is true—that the sample result occurred by chance. For example, a misguided researcher might say that because the  p  value is .02, there is only a 2% chance that the result is due to chance and a 98% chance that it reflects a real relationship in the population. But this is incorrect . The  p  value is really the probability of a result at least as extreme as the sample result  if  the null hypothesis  were  true. So a  p  value of .02 means that if the null hypothesis were true, a sample result this extreme would occur only 2% of the time.

You can avoid this misunderstanding by remembering that the  p  value is not the probability that any particular  hypothesis  is true or false. Instead, it is the probability of obtaining the  sample result  if the null hypothesis were true.

Role of Sample Size and Relationship Strength

Recall that null hypothesis testing involves answering the question, “If the null hypothesis were true, what is the probability of a sample result as extreme as this one?” In other words, “What is the  p  value?” It can be helpful to see that the answer to this question depends on just two considerations: the strength of the relationship and the size of the sample. Specifically, the stronger the sample relationship and the larger the sample, the less likely the result would be if the null hypothesis were true. That is, the lower the  p  value. This should make sense. Imagine a study in which a sample of 500 women is compared with a sample of 500 men in terms of some psychological characteristic, and Cohen’s  d  is a strong 0.50. If there were really no sex difference in the population, then a result this strong based on such a large sample should seem highly unlikely. Now imagine a similar study in which a sample of three women is compared with a sample of three men, and Cohen’s  d  is a weak 0.10. If there were no sex difference in the population, then a relationship this weak based on such a small sample should seem likely. And this is precisely why the null hypothesis would be rejected in the first example and retained in the second.

Of course, sometimes the result can be weak and the sample large, or the result can be strong and the sample small. In these cases, the two considerations trade off against each other so that a weak result can be statistically significant if the sample is large enough and a strong relationship can be statistically significant even if the sample is small. Table 13.1 shows roughly how relationship strength and sample size combine to determine whether a sample result is statistically significant. The columns of the table represent the three levels of relationship strength: weak, medium, and strong. The rows represent four sample sizes that can be considered small, medium, large, and extra large in the context of psychological research. Thus each cell in the table represents a combination of relationship strength and sample size. If a cell contains the word  Yes , then this combination would be statistically significant for both Cohen’s  d  and Pearson’s  r . If it contains the word  No , then it would not be statistically significant for either. There is one cell where the decision for  d  and  r  would be different and another where it might be different depending on some additional considerations, which are discussed in Section 13.2 “Some Basic Null Hypothesis Tests”

Table 13.1 How Relationship Strength and Sample Size Combine to Determine Whether a Result Is Statistically Significant
Sample Size Weak relationship Medium-strength relationship Strong relationship
Small (  = 20) No No  = Maybe

 = Yes

Medium (  = 50) No Yes Yes
Large (  = 100)  = Yes

 = No

Yes Yes
Extra large (  = 500) Yes Yes Yes

Although Table 13.1 provides only a rough guideline, it shows very clearly that weak relationships based on medium or small samples are never statistically significant and that strong relationships based on medium or larger samples are always statistically significant. If you keep this lesson in mind, you will often know whether a result is statistically significant based on the descriptive statistics alone. It is extremely useful to be able to develop this kind of intuitive judgment. One reason is that it allows you to develop expectations about how your formal null hypothesis tests are going to come out, which in turn allows you to detect problems in your analyses. For example, if your sample relationship is strong and your sample is medium, then you would expect to reject the null hypothesis. If for some reason your formal null hypothesis test indicates otherwise, then you need to double-check your computations and interpretations. A second reason is that the ability to make this kind of intuitive judgment is an indication that you understand the basic logic of this approach in addition to being able to do the computations.

Statistical Significance Versus Practical Significance

Table 13.1 illustrates another extremely important point. A statistically significant result is not necessarily a strong one. Even a very weak result can be statistically significant if it is based on a large enough sample. This is closely related to Janet Shibley Hyde’s argument about sex differences (Hyde, 2007) [2] . The differences between women and men in mathematical problem solving and leadership ability are statistically significant. But the word  significant  can cause people to interpret these differences as strong and important—perhaps even important enough to influence the college courses they take or even who they vote for. As we have seen, however, these statistically significant differences are actually quite weak—perhaps even “trivial.”

This is why it is important to distinguish between the  statistical  significance of a result and the  practical  significance of that result.  Practical significance refers to the importance or usefulness of the result in some real-world context. Many sex differences are statistically significant—and may even be interesting for purely scientific reasons—but they are not practically significant. In clinical practice, this same concept is often referred to as “clinical significance.” For example, a study on a new treatment for social phobia might show that it produces a statistically significant positive effect. Yet this effect still might not be strong enough to justify the time, effort, and other costs of putting it into practice—especially if easier and cheaper treatments that work almost as well already exist. Although statistically significant, this result would be said to lack practical or clinical significance.

Key Takeaways

  • Null hypothesis testing is a formal approach to deciding whether a statistical relationship in a sample reflects a real relationship in the population or is just due to chance.
  • The logic of null hypothesis testing involves assuming that the null hypothesis is true, finding how likely the sample result would be if this assumption were correct, and then making a decision. If the sample result would be unlikely if the null hypothesis were true, then it is rejected in favour of the alternative hypothesis. If it would not be unlikely, then the null hypothesis is retained.
  • The probability of obtaining the sample result if the null hypothesis were true (the  p  value) is based on two considerations: relationship strength and sample size. Reasonable judgments about whether a sample relationship is statistically significant can often be made by quickly considering these two factors.
  • Statistical significance is not the same as relationship strength or importance. Even weak relationships can be statistically significant if the sample size is large enough. It is important to consider relationship strength and the practical significance of a result in addition to its statistical significance.
  • Discussion: Imagine a study showing that people who eat more broccoli tend to be happier. Explain for someone who knows nothing about statistics why the researchers would conduct a null hypothesis test.
  • The correlation between two variables is  r  = −.78 based on a sample size of 137.
  • The mean score on a psychological characteristic for women is 25 ( SD  = 5) and the mean score for men is 24 ( SD  = 5). There were 12 women and 10 men in this study.
  • In a memory experiment, the mean number of items recalled by the 40 participants in Condition A was 0.50 standard deviations greater than the mean number recalled by the 40 participants in Condition B.
  • In another memory experiment, the mean scores for participants in Condition A and Condition B came out exactly the same!
  • A student finds a correlation of  r  = .04 between the number of units the students in his research methods class are taking and the students’ level of stress.

Long Descriptions

“Null Hypothesis” long description: A comic depicting a man and a woman talking in the foreground. In the background is a child working at a desk. The man says to the woman, “I can’t believe schools are still teaching kids about the null hypothesis. I remember reading a big study that conclusively disproved it years ago.” [Return to “Null Hypothesis”]

“Conditional Risk” long description: A comic depicting two hikers beside a tree during a thunderstorm. A bolt of lightning goes “crack” in the dark sky as thunder booms. One of the hikers says, “Whoa! We should get inside!” The other hiker says, “It’s okay! Lightning only kills about 45 Americans a year, so the chances of dying are only one in 7,000,000. Let’s go on!” The comic’s caption says, “The annual death rate among people who know that statistic is one in six.” [Return to “Conditional Risk”]

Media Attributions

  • Null Hypothesis by XKCD  CC BY-NC (Attribution NonCommercial)
  • Conditional Risk by XKCD  CC BY-NC (Attribution NonCommercial)
  • Cohen, J. (1994). The world is round: p < .05. American Psychologist, 49 , 997–1003. ↵
  • Hyde, J. S. (2007). New directions in the study of gender similarities and differences. Current Directions in Psychological Science, 16 , 259–263. ↵

Values in a population that correspond to variables measured in a study.

The random variability in a statistic from sample to sample.

A formal approach to deciding between two interpretations of a statistical relationship in a sample.

The idea that there is no relationship in the population and that the relationship in the sample reflects only sampling error.

The idea that there is a relationship in the population and that the relationship in the sample reflects this relationship in the population.

When the relationship found in the sample would be extremely unlikely, the idea that the relationship occurred “by chance” is rejected.

When the relationship found in the sample is likely to have occurred by chance, the null hypothesis is not rejected.

The probability that, if the null hypothesis were true, the result found in the sample would occur.

How low the p value must be before the sample result is considered unlikely in null hypothesis testing.

When there is less than a 5% chance of a result as extreme as the sample result occurring and the null hypothesis is rejected.

Research Methods in Psychology - 2nd Canadian Edition Copyright © 2015 by Paul C. Price, Rajiv Jhangiani, & I-Chant A. Chiang is licensed under a Creative Commons Attribution-NonCommercial-ShareAlike 4.0 International License , except where otherwise noted.

Share This Book

is rejecting null hypothesis good

Support or Reject Null Hypothesis in Easy Steps

What does it mean to reject the null hypothesis.

  • General Situations: P Value
  • P Value Guidelines
  • A Proportion
  • A Proportion (second example)

In many statistical tests, you’ll want to either reject or support the null hypothesis . For elementary statistics students, the term can be a tricky term to grasp, partly because the name “null hypothesis” doesn’t make it clear about what the null hypothesis actually is!

The null hypothesis can be thought of as a nullifiable hypothesis. That means you can nullify it, or reject it. What happens if you reject the null hypothesis? It gets replaced with the alternate hypothesis, which is what you think might actually be true about a situation. For example, let’s say you think that a certain drug might be responsible for a spate of recent heart attacks. The drug company thinks the drug is safe. The null hypothesis is always the accepted hypothesis; in this example, the drug is on the market, people are using it, and it’s generally accepted to be safe. Therefore, the null hypothesis is that the drug is safe. The alternate hypothesis — the one you want to replace the null hypothesis, is that the drug isn’t safe. Rejecting the null hypothesis in this case means that you will have to prove that the drug is not safe.

reject the null hypothesis

To reject the null hypothesis, perform the following steps:

Step 1: State the null hypothesis. When you state the null hypothesis, you also have to state the alternate hypothesis. Sometimes it is easier to state the alternate hypothesis first, because that’s the researcher’s thoughts about the experiment. How to state the null hypothesis (opens in a new window).

Step 2: Support or reject the null hypothesis . Several methods exist, depending on what kind of sample data you have. For example, you can use the P-value method. For a rundown on all methods, see: Support or reject the null hypothesis.

If you are able to reject the null hypothesis in Step 2, you can replace it with the alternate hypothesis.

That’s it!

When to Reject the Null hypothesis

Basically, you reject the null hypothesis when your test value falls into the rejection region . There are four main ways you’ll compute test values and either support or reject your null hypothesis. Which method you choose depends mainly on if you have a proportion or a p-value .

support or reject null hypothesis

Support or Reject the Null Hypothesis: Steps

Click the link the skip to the situation you need to support or reject null hypothesis for: General Situations: P Value P Value Guidelines A Proportion A Proportion (second example)

Support or Reject Null Hypothesis with a P Value

If you have a P-value , or are asked to find a p-value, follow these instructions to support or reject the null hypothesis. This method works if you are given an alpha level and if you are not given an alpha level. If you are given a confidence level , just subtract from 1 to get the alpha level. See: How to calculate an alpha level .

Step 1: State the null hypothesis and the alternate hypothesis (“the claim”). If you aren’t sure how to do this, follow this link for How To State the Null and Alternate Hypothesis .

Step 2: Find the critical value . We’re dealing with a normally distributed population, so the critical value is a z-score . Use the following formula to find the z-score .

null hypothesis z formula

Click here if you want easy, step-by-step instructions for solving this formula.

Step 4: Find the P-Value by looking up your answer from step 3 in the z-table . To get the p-value, subtract the area from 1. For example, if your area is .990 then your p-value is 1-.9950 = 0.005. Note: for a two-tailed test , you’ll need to halve this amount to get the p-value in one tail.

Step 5: Compare your answer from step 4 with the α value given in the question. Should you support or reject the null hypothesis? If step 7 is less than or equal to α, reject the null hypothesis, otherwise do not reject it.

P-Value Guidelines

Use these general guidelines to decide if you should reject or keep the null:

If p value > .10 → “not significant ” If p value ≤ .10 → “marginally significant” If p value ≤ .05 → “significant” If p value ≤ .01 → “highly significant.”

Back to Top

Support or Reject Null Hypothesis for a Proportion

Sometimes, you’ll be given a proportion of the population or a percentage and asked to support or reject null hypothesis. In this case you can’t compute a test value by calculating a z-score (you need actual numbers for that), so we use a slightly different technique.

Example question: A researcher claims that Democrats will win the next election. 4300 voters were polled; 2200 said they would vote Democrat. Decide if you should support or reject null hypothesis. Is there enough evidence at α=0.05 to support this claim?

Step 1: State the null hypothesis and the alternate hypothesis (“the claim”) . H o :p ≤ 0.5 H 1 :p > .5

phat

Step 3: Use the following formula to calculate your test value.

test value with a proportion

Where: Phat is calculated in Step 2 P the null hypothesis p value (.05) Q is 1 – p

The z-score is: .512 – .5 / √(.5(.5) / 4300)) = 1.57

Step 4: Look up Step 3 in the z-table to get .9418.

Step 5: Calculate your p-value by subtracting Step 4 from 1. 1-.9418 = .0582

Step 6: Compare your answer from step 5 with the α value given in the question . Support or reject the null hypothesis? If step 5 is less than α, reject the null hypothesis, otherwise do not reject it. In this case, .582 (5.82%) is not less than our α, so we do not reject the null hypothesis.

Support or Reject Null Hypothesis for a Proportion: Second example

Example question: A researcher claims that more than 23% of community members go to church regularly. In a recent survey, 126 out of 420 people stated they went to church regularly. Is there enough evidence at α = 0.05 to support this claim? Use the P-Value method to support or reject null hypothesis.

Step 1: State the null hypothesis and the alternate hypothesis (“the claim”) . H o :p ≤ 0.23; H 1 :p > 0.23 (claim)

Step 3: Find ‘p’ by converting the stated claim to a decimal: 23% = 0.23. Also, find ‘q’ by subtracting ‘p’ from 1: 1 – 0.23 = 0.77.

Step 4: Use the following formula to calculate your test value.

HYPOTHESIS test value with a proportion

If formulas confuse you, this is asking you to:

  • Multiply p and q together, then divide by the number in the random sample. (0.23 x 0.77) / 420 = 0.00042
  • Take the square root of your answer to 2 . √( 0.1771) = 0. 0205
  • Divide your answer to 1. by your answer in 3. 0.07 / 0. 0205 = 3.41

Step 5: Find the P-Value by looking up your answer from step 5 in the z-table . The z-score for 3.41 is .4997. Subtract from 0.500: 0.500-.4997 = 0.003.

Step 6: Compare your P-value to α . Support or reject null hypothesis? If the P-value is less, reject the null hypothesis. If the P-value is more, keep the null hypothesis. 0.003 < 0.05, so we have enough evidence to reject the null hypothesis and accept the claim.

Note: In Step 5, I’m using the z-table on this site to solve this problem. Most textbooks have the right of z-table . If you’re seeing .9997 as an answer in your textbook table, then your textbook has a “whole z” table, in which case don’t subtract from .5, subtract from 1. 1-.9997 = 0.003.

Check out our Youtube channel for video tips!

Everitt, B. S.; Skrondal, A. (2010), The Cambridge Dictionary of Statistics , Cambridge University Press. Gonick, L. (1993). The Cartoon Guide to Statistics . HarperPerennial.

is rejecting null hypothesis good

User Preferences

Content preview.

Arcu felis bibendum ut tristique et egestas quis:

  • Ut enim ad minim veniam, quis nostrud exercitation ullamco laboris
  • Duis aute irure dolor in reprehenderit in voluptate
  • Excepteur sint occaecat cupidatat non proident

Keyboard Shortcuts

S.3.2 hypothesis testing (p-value approach).

The P -value approach involves determining "likely" or "unlikely" by determining the probability — assuming the null hypothesis was true — of observing a more extreme test statistic in the direction of the alternative hypothesis than the one observed. If the P -value is small, say less than (or equal to) \(\alpha\), then it is "unlikely." And, if the P -value is large, say more than \(\alpha\), then it is "likely."

If the P -value is less than (or equal to) \(\alpha\), then the null hypothesis is rejected in favor of the alternative hypothesis. And, if the P -value is greater than \(\alpha\), then the null hypothesis is not rejected.

Specifically, the four steps involved in using the P -value approach to conducting any hypothesis test are:

  • Specify the null and alternative hypotheses.
  • Using the sample data and assuming the null hypothesis is true, calculate the value of the test statistic. Again, to conduct the hypothesis test for the population mean μ , we use the t -statistic \(t^*=\frac{\bar{x}-\mu}{s/\sqrt{n}}\) which follows a t -distribution with n - 1 degrees of freedom.
  • Using the known distribution of the test statistic, calculate the P -value : "If the null hypothesis is true, what is the probability that we'd observe a more extreme test statistic in the direction of the alternative hypothesis than we did?" (Note how this question is equivalent to the question answered in criminal trials: "If the defendant is innocent, what is the chance that we'd observe such extreme criminal evidence?")
  • Set the significance level, \(\alpha\), the probability of making a Type I error to be small — 0.01, 0.05, or 0.10. Compare the P -value to \(\alpha\). If the P -value is less than (or equal to) \(\alpha\), reject the null hypothesis in favor of the alternative hypothesis. If the P -value is greater than \(\alpha\), do not reject the null hypothesis.

Example S.3.2.1

Mean gpa section  .

In our example concerning the mean grade point average, suppose that our random sample of n = 15 students majoring in mathematics yields a test statistic t * equaling 2.5. Since n = 15, our test statistic t * has n - 1 = 14 degrees of freedom. Also, suppose we set our significance level α at 0.05 so that we have only a 5% chance of making a Type I error.

Right Tailed

The P -value for conducting the right-tailed test H 0 : μ = 3 versus H A : μ > 3 is the probability that we would observe a test statistic greater than t * = 2.5 if the population mean \(\mu\) really were 3. Recall that probability equals the area under the probability curve. The P -value is therefore the area under a t n - 1 = t 14 curve and to the right of the test statistic t * = 2.5. It can be shown using statistical software that the P -value is 0.0127. The graph depicts this visually.

t-distrbution graph showing the right tail beyond a t value of 2.5

The P -value, 0.0127, tells us it is "unlikely" that we would observe such an extreme test statistic t * in the direction of H A if the null hypothesis were true. Therefore, our initial assumption that the null hypothesis is true must be incorrect. That is, since the P -value, 0.0127, is less than \(\alpha\) = 0.05, we reject the null hypothesis H 0 : μ = 3 in favor of the alternative hypothesis H A : μ > 3.

Note that we would not reject H 0 : μ = 3 in favor of H A : μ > 3 if we lowered our willingness to make a Type I error to \(\alpha\) = 0.01 instead, as the P -value, 0.0127, is then greater than \(\alpha\) = 0.01.

Left Tailed

In our example concerning the mean grade point average, suppose that our random sample of n = 15 students majoring in mathematics yields a test statistic t * instead of equaling -2.5. The P -value for conducting the left-tailed test H 0 : μ = 3 versus H A : μ < 3 is the probability that we would observe a test statistic less than t * = -2.5 if the population mean μ really were 3. The P -value is therefore the area under a t n - 1 = t 14 curve and to the left of the test statistic t* = -2.5. It can be shown using statistical software that the P -value is 0.0127. The graph depicts this visually.

t distribution graph showing left tail below t value of -2.5

The P -value, 0.0127, tells us it is "unlikely" that we would observe such an extreme test statistic t * in the direction of H A if the null hypothesis were true. Therefore, our initial assumption that the null hypothesis is true must be incorrect. That is, since the P -value, 0.0127, is less than α = 0.05, we reject the null hypothesis H 0 : μ = 3 in favor of the alternative hypothesis H A : μ < 3.

Note that we would not reject H 0 : μ = 3 in favor of H A : μ < 3 if we lowered our willingness to make a Type I error to α = 0.01 instead, as the P -value, 0.0127, is then greater than \(\alpha\) = 0.01.

In our example concerning the mean grade point average, suppose again that our random sample of n = 15 students majoring in mathematics yields a test statistic t * instead of equaling -2.5. The P -value for conducting the two-tailed test H 0 : μ = 3 versus H A : μ ≠ 3 is the probability that we would observe a test statistic less than -2.5 or greater than 2.5 if the population mean μ really was 3. That is, the two-tailed test requires taking into account the possibility that the test statistic could fall into either tail (hence the name "two-tailed" test). The P -value is, therefore, the area under a t n - 1 = t 14 curve to the left of -2.5 and to the right of 2.5. It can be shown using statistical software that the P -value is 0.0127 + 0.0127, or 0.0254. The graph depicts this visually.

t-distribution graph of two tailed probability for t values of -2.5 and 2.5

Note that the P -value for a two-tailed test is always two times the P -value for either of the one-tailed tests. The P -value, 0.0254, tells us it is "unlikely" that we would observe such an extreme test statistic t * in the direction of H A if the null hypothesis were true. Therefore, our initial assumption that the null hypothesis is true must be incorrect. That is, since the P -value, 0.0254, is less than α = 0.05, we reject the null hypothesis H 0 : μ = 3 in favor of the alternative hypothesis H A : μ ≠ 3.

Note that we would not reject H 0 : μ = 3 in favor of H A : μ ≠ 3 if we lowered our willingness to make a Type I error to α = 0.01 instead, as the P -value, 0.0254, is then greater than \(\alpha\) = 0.01.

Now that we have reviewed the critical value and P -value approach procedures for each of the three possible hypotheses, let's look at three new examples — one of a right-tailed test, one of a left-tailed test, and one of a two-tailed test.

The good news is that, whenever possible, we will take advantage of the test statistics and P -values reported in statistical software, such as Minitab, to conduct our hypothesis tests in this course.

Rejecting the Null Hypothesis Using Confidence Intervals

  • Tech Trends

Rejecting the Null Hypothesis Using Confidence Intervals

In an introductory statistics class, there are three main topics that are taught: descriptive statistics and data visualizations, probability and sampling distributions, and statistical inference. Within statistical inference, there are two key methods of statistical inference that are taught, viz. confidence intervals and hypothesis testing . While these two methods are always taught when learning data science and related fields, it is rare that the relationship between these two methods is properly elucidated.

In this article, we’ll begin by defining and describing each method of statistical inference in turn and along the way, state what statistical inference is, and perhaps more importantly, what it isn’t. Then we’ll describe the relationship between the two. While it is typically the case that confidence intervals are taught before hypothesis testing when learning statistics, we’ll begin with the latter since it will allow us to define statistical significance.

Hypothesis Tests

The purpose of a hypothesis test is to answer whether random chance might be responsible for an observed effect. Hypothesis tests use sample statistics to test a hypothesis about population parameters. The null hypothesis, H 0 , is a statement that represents the assumed status quo regarding a variable or variables and it is always about a population characteristic. Some of the ways the null hypothesis is typically glossed are: the population variable is equal to a particular value or there is no difference between the population variables . For example:

  • H 0 : μ = 61 in (The mean height of the population of American men is 69 inches)
  • H 0 : p 1 -p 2 = 0 (The difference in the population proportions of women who prefer football over baseball and the population proportion of men who prefer football over baseball is 0.)

Note that the null hypothesis always has the equal sign.

The alternative hypothesis, denoted either H 1 or H a , is the statement that is opposed to the null hypothesis (e.g., the population variable is not equal to a particular value  or there is a difference between the population variables ):

  • H 1 : μ > 61 im (The mean height of the population of American men is greater than 69 inches.)
  • H 1 : p 1 -p 2 ≠ 0 (The difference in the population proportions of women who prefer football over baseball and the population proportion of men who prefer football over baseball is not 0.)

The alternative hypothesis is typically the claim that the researcher hopes to show and it always contains the strict inequality symbols (‘<’ left-sided or left-tailed, ‘≠’ two-sided or two-tailed, and ‘>’ right-sided or right-tailed).

When carrying out a test of H 0 vs. H 1 , the null hypothesis H 0 will be rejected in favor of the alternative hypothesis only if the sample provides convincing evidence that H 0 is false. As such, a statistical hypothesis test is only capable of demonstrating strong support for the alternative hypothesis by rejecting the null hypothesis.

When the null hypothesis is not rejected, it does not mean that there is strong support for the null hypothesis (since it was assumed to be true); rather, only that there is not convincing evidence against the null hypothesis. As such, we never use the phrase “accept the null hypothesis.”

In the classical method of performing hypothesis testing, one would have to find what is called the test statistic and use a table to find the corresponding probability. Happily, due to the advancement of technology, one can use Python (as is done in the Flatiron’s Data Science Bootcamp ) and get the required value directly using a Python library like stats models . This is the p-value , which is short for the probability value.

The p-value is a measure of inconsistency between the hypothesized value for a population characteristic and the observed sample. The p -value is the probability, under the assumption the null hypothesis is true, of obtaining a test statistic value that is a measure of inconsistency between the null hypothesis and the data. If the p -value is less than or equal to the probability of the Type I error, then we can reject the null hypothesis and we have sufficient evidence to support the alternative hypothesis.

Typically the probability of a Type I error ɑ, more commonly known as the level of significance , is set to be 0.05, but it is often prudent to have it set to values less than that such as 0.01 or 0.001. Thus, if p -value ≤ ɑ, then we reject the null hypothesis and we interpret this as saying there is a statistically significant difference between the sample and the population. So if the p -value=0.03 ≤ 0.05 = ɑ, then we would reject the null hypothesis and so have statistical significance, whereas if p -value=0.08 ≥ 0.05 = ɑ, then we would fail to reject the null hypothesis and there would not be statistical significance.

Confidence Intervals

The other primary form of statistical inference are confidence intervals. While hypothesis tests are concerned with testing a claim, the purpose of a confidence interval is to estimate an unknown population characteristic. A confidence interval is an interval of plausible values for a population characteristic. They are constructed so that we have a chosen level of confidence that the actual value of the population characteristic will be between the upper and lower endpoints of the open interval.

The structure of an individual confidence interval is the sample estimate of the variable of interest margin of error. The margin of error is the product of a multiplier value and the standard error, s.e., which is based on the standard deviation and the sample size. The multiplier is where the probability, of level of confidence, is introduced into the formula.

The confidence level is the success rate of the method used to construct a confidence interval. A confidence interval estimating the proportion of American men who state they are an avid fan of the NFL could be (0.40, 0.60) with a 95% level of confidence. The level of confidence is not the probability that that population characteristic is in the confidence interval, but rather refers to the method that is used to construct the confidence interval.

For example, a 95% confidence interval would be interpreted as if one constructed 100 confidence intervals, then 95 of them would contain the true population characteristic. 

Errors and Power

A Type I error, or a false positive, is the error of finding a difference that is not there, so it is the probability of incorrectly rejecting a true null hypothesis is ɑ, where ɑ is the level of significance. It follows that the probability of correctly failing to reject a true null hypothesis is the complement of it, viz. 1 – ɑ. For a particular hypothesis test, if ɑ = 0.05, then its complement would be 0.95 or 95%.

While we are not going to expand on these ideas, we note the following two related probabilities. A Type II error, or false negative, is the probability of failing to reject a false null hypothesis where the probability of a type II error is β and the power is the probability of correctly rejecting a false null hypothesis where power = 1 – β. In common statistical practice, one typically only speaks of the level of significance and the power.

The following table summarizes these ideas , where the column headers refer to what is actually the case, but is unknown. (If the truth or falsity of the null value was truly known, we wouldn’t have to do statistics.)

is rejecting null hypothesis good

Hypothesis Tests and Confidence Intervals

Since hypothesis tests and confidence intervals are both methods of statistical inference, then it is reasonable to wonder if they are equivalent in some way. The answer is yes, which means that we can perform hypothesis testing using confidence intervals.

Returning to the example where we have an estimate of the proportion of American men that are avid fans of the NFL, we had (0.40, 0.60) at a 95% confidence level. As a hypothesis test, we could have the alternative hypothesis as H 1 ≠ 0.51. Since the null value of 0.51 lies within the confidence interval, then we would fail to reject the null hypothesis at ɑ = 0.05.

On the other hand, if H 1 ≠ 0.61, then since 0.61 is not in the confidence interval we can reject the null hypothesis at ɑ = 0.05. Note that the confidence level of 95% and the level of significance at ɑ = 0.05 = 5%  are complements, which is the “H o is True” column in the above table.

In general, one can reject the null hypothesis given a null value and a confidence interval for a two-sided test if the null value is not in the confidence interval where the confidence level and level of significance are complements. For one-sided tests, one can still perform a hypothesis test with the confidence level and null value. Not only is there an added layer of complexity for this equivalence, it is the best practice to perform two-sided hypothesis tests since one is not prejudicing the direction of the alternative.

In this discussion of hypothesis testing and confidence intervals, we not only understand when these two methods of statistical inference can be equivalent, but now have a deeper understanding of statistical significance itself and therefore, statistical inference.

Learn More About Data Science at Flatiron

The curriculum in our Data Science Bootcamp incorporates the latest technologies, including artificial intelligence (AI) tools. Download the syllabus to see what you can learn, or book a 10-minute call with Admissions to learn about full-time and part-time attendance opportunities.

is rejecting null hypothesis good

About Brendan Patrick Purdy

Brendan is the senior curriculum developer for data science at the Flatiron School. He holds degrees in mathematics, data science, and philosophy, and enjoys modeling neural networks with the Python library TensorFlow.

Related Resources

is rejecting null hypothesis good

Data Science 101 | Crack the Code with Gemini: Using AI to Build a Linear Regression Model

is rejecting null hypothesis good

Quantifying Rafael Nadal’s Dominance with French Open Data

The Art of Data Exploration

The Art of Data Exploration

Privacy overview.

CookieDurationDescription
cookielawinfo-checkbox-analytics11 monthsThis cookie is set by GDPR Cookie Consent plugin. The cookie is used to store the user consent for the cookies in the category "Analytics".
cookielawinfo-checkbox-functional11 monthsThe cookie is set by GDPR cookie consent to record the user consent for the cookies in the category "Functional".
cookielawinfo-checkbox-necessary11 monthsThis cookie is set by GDPR Cookie Consent plugin. The cookies is used to store the user consent for the cookies in the category "Necessary".
cookielawinfo-checkbox-others11 monthsThis cookie is set by GDPR Cookie Consent plugin. The cookie is used to store the user consent for the cookies in the category "Other.
cookielawinfo-checkbox-performance11 monthsThis cookie is set by GDPR Cookie Consent plugin. The cookie is used to store the user consent for the cookies in the category "Performance".
viewed_cookie_policy11 monthsThe cookie is set by the GDPR Cookie Consent plugin and is used to store whether or not user has consented to the use of cookies. It does not store any personal data.

P-Value And Statistical Significance: What It Is & Why It Matters

Saul McLeod, PhD

Editor-in-Chief for Simply Psychology

BSc (Hons) Psychology, MRes, PhD, University of Manchester

Saul McLeod, PhD., is a qualified psychology teacher with over 18 years of experience in further and higher education. He has been published in peer-reviewed journals, including the Journal of Clinical Psychology.

Learn about our Editorial Process

Olivia Guy-Evans, MSc

Associate Editor for Simply Psychology

BSc (Hons) Psychology, MSc Psychology of Education

Olivia Guy-Evans is a writer and associate editor for Simply Psychology. She has previously worked in healthcare and educational sectors.

On This Page:

The p-value in statistics quantifies the evidence against a null hypothesis. A low p-value suggests data is inconsistent with the null, potentially favoring an alternative hypothesis. Common significance thresholds are 0.05 or 0.01.

P-Value Explained in Normal Distribution

Hypothesis testing

When you perform a statistical test, a p-value helps you determine the significance of your results in relation to the null hypothesis.

The null hypothesis (H0) states no relationship exists between the two variables being studied (one variable does not affect the other). It states the results are due to chance and are not significant in supporting the idea being investigated. Thus, the null hypothesis assumes that whatever you try to prove did not happen.

The alternative hypothesis (Ha or H1) is the one you would believe if the null hypothesis is concluded to be untrue.

The alternative hypothesis states that the independent variable affected the dependent variable, and the results are significant in supporting the theory being investigated (i.e., the results are not due to random chance).

What a p-value tells you

A p-value, or probability value, is a number describing how likely it is that your data would have occurred by random chance (i.e., that the null hypothesis is true).

The level of statistical significance is often expressed as a p-value between 0 and 1.

The smaller the p -value, the less likely the results occurred by random chance, and the stronger the evidence that you should reject the null hypothesis.

Remember, a p-value doesn’t tell you if the null hypothesis is true or false. It just tells you how likely you’d see the data you observed (or more extreme data) if the null hypothesis was true. It’s a piece of evidence, not a definitive proof.

Example: Test Statistic and p-Value

Suppose you’re conducting a study to determine whether a new drug has an effect on pain relief compared to a placebo. If the new drug has no impact, your test statistic will be close to the one predicted by the null hypothesis (no difference between the drug and placebo groups), and the resulting p-value will be close to 1. It may not be precisely 1 because real-world variations may exist. Conversely, if the new drug indeed reduces pain significantly, your test statistic will diverge further from what’s expected under the null hypothesis, and the p-value will decrease. The p-value will never reach zero because there’s always a slim possibility, though highly improbable, that the observed results occurred by random chance.

P-value interpretation

The significance level (alpha) is a set probability threshold (often 0.05), while the p-value is the probability you calculate based on your study or analysis.

A p-value less than or equal to your significance level (typically ≤ 0.05) is statistically significant.

A p-value less than or equal to a predetermined significance level (often 0.05 or 0.01) indicates a statistically significant result, meaning the observed data provide strong evidence against the null hypothesis.

This suggests the effect under study likely represents a real relationship rather than just random chance.

For instance, if you set α = 0.05, you would reject the null hypothesis if your p -value ≤ 0.05. 

It indicates strong evidence against the null hypothesis, as there is less than a 5% probability the null is correct (and the results are random).

Therefore, we reject the null hypothesis and accept the alternative hypothesis.

Example: Statistical Significance

Upon analyzing the pain relief effects of the new drug compared to the placebo, the computed p-value is less than 0.01, which falls well below the predetermined alpha value of 0.05. Consequently, you conclude that there is a statistically significant difference in pain relief between the new drug and the placebo.

What does a p-value of 0.001 mean?

A p-value of 0.001 is highly statistically significant beyond the commonly used 0.05 threshold. It indicates strong evidence of a real effect or difference, rather than just random variation.

Specifically, a p-value of 0.001 means there is only a 0.1% chance of obtaining a result at least as extreme as the one observed, assuming the null hypothesis is correct.

Such a small p-value provides strong evidence against the null hypothesis, leading to rejecting the null in favor of the alternative hypothesis.

A p-value more than the significance level (typically p > 0.05) is not statistically significant and indicates strong evidence for the null hypothesis.

This means we retain the null hypothesis and reject the alternative hypothesis. You should note that you cannot accept the null hypothesis; we can only reject it or fail to reject it.

Note : when the p-value is above your threshold of significance,  it does not mean that there is a 95% probability that the alternative hypothesis is true.

One-Tailed Test

Probability and statistical significance in ab testing. Statistical significance in a b experiments

Two-Tailed Test

statistical significance two tailed

How do you calculate the p-value ?

Most statistical software packages like R, SPSS, and others automatically calculate your p-value. This is the easiest and most common way.

Online resources and tables are available to estimate the p-value based on your test statistic and degrees of freedom.

These tables help you understand how often you would expect to see your test statistic under the null hypothesis.

Understanding the Statistical Test:

Different statistical tests are designed to answer specific research questions or hypotheses. Each test has its own underlying assumptions and characteristics.

For example, you might use a t-test to compare means, a chi-squared test for categorical data, or a correlation test to measure the strength of a relationship between variables.

Be aware that the number of independent variables you include in your analysis can influence the magnitude of the test statistic needed to produce the same p-value.

This factor is particularly important to consider when comparing results across different analyses.

Example: Choosing a Statistical Test

If you’re comparing the effectiveness of just two different drugs in pain relief, a two-sample t-test is a suitable choice for comparing these two groups. However, when you’re examining the impact of three or more drugs, it’s more appropriate to employ an Analysis of Variance ( ANOVA) . Utilizing multiple pairwise comparisons in such cases can lead to artificially low p-values and an overestimation of the significance of differences between the drug groups.

How to report

A statistically significant result cannot prove that a research hypothesis is correct (which implies 100% certainty).

Instead, we may state our results “provide support for” or “give evidence for” our research hypothesis (as there is still a slight probability that the results occurred by chance and the null hypothesis was correct – e.g., less than 5%).

Example: Reporting the results

In our comparison of the pain relief effects of the new drug and the placebo, we observed that participants in the drug group experienced a significant reduction in pain ( M = 3.5; SD = 0.8) compared to those in the placebo group ( M = 5.2; SD  = 0.7), resulting in an average difference of 1.7 points on the pain scale (t(98) = -9.36; p < 0.001).

The 6th edition of the APA style manual (American Psychological Association, 2010) states the following on the topic of reporting p-values:

“When reporting p values, report exact p values (e.g., p = .031) to two or three decimal places. However, report p values less than .001 as p < .001.

The tradition of reporting p values in the form p < .10, p < .05, p < .01, and so forth, was appropriate in a time when only limited tables of critical values were available.” (p. 114)

  • Do not use 0 before the decimal point for the statistical value p as it cannot equal 1. In other words, write p = .001 instead of p = 0.001.
  • Please pay attention to issues of italics ( p is always italicized) and spacing (either side of the = sign).
  • p = .000 (as outputted by some statistical packages such as SPSS) is impossible and should be written as p < .001.
  • The opposite of significant is “nonsignificant,” not “insignificant.”

Why is the p -value not enough?

A lower p-value  is sometimes interpreted as meaning there is a stronger relationship between two variables.

However, statistical significance means that it is unlikely that the null hypothesis is true (less than 5%).

To understand the strength of the difference between the two groups (control vs. experimental) a researcher needs to calculate the effect size .

When do you reject the null hypothesis?

In statistical hypothesis testing, you reject the null hypothesis when the p-value is less than or equal to the significance level (α) you set before conducting your test. The significance level is the probability of rejecting the null hypothesis when it is true. Commonly used significance levels are 0.01, 0.05, and 0.10.

Remember, rejecting the null hypothesis doesn’t prove the alternative hypothesis; it just suggests that the alternative hypothesis may be plausible given the observed data.

The p -value is conditional upon the null hypothesis being true but is unrelated to the truth or falsity of the alternative hypothesis.

What does p-value of 0.05 mean?

If your p-value is less than or equal to 0.05 (the significance level), you would conclude that your result is statistically significant. This means the evidence is strong enough to reject the null hypothesis in favor of the alternative hypothesis.

Are all p-values below 0.05 considered statistically significant?

No, not all p-values below 0.05 are considered statistically significant. The threshold of 0.05 is commonly used, but it’s just a convention. Statistical significance depends on factors like the study design, sample size, and the magnitude of the observed effect.

A p-value below 0.05 means there is evidence against the null hypothesis, suggesting a real effect. However, it’s essential to consider the context and other factors when interpreting results.

Researchers also look at effect size and confidence intervals to determine the practical significance and reliability of findings.

How does sample size affect the interpretation of p-values?

Sample size can impact the interpretation of p-values. A larger sample size provides more reliable and precise estimates of the population, leading to narrower confidence intervals.

With a larger sample, even small differences between groups or effects can become statistically significant, yielding lower p-values. In contrast, smaller sample sizes may not have enough statistical power to detect smaller effects, resulting in higher p-values.

Therefore, a larger sample size increases the chances of finding statistically significant results when there is a genuine effect, making the findings more trustworthy and robust.

Can a non-significant p-value indicate that there is no effect or difference in the data?

No, a non-significant p-value does not necessarily indicate that there is no effect or difference in the data. It means that the observed data do not provide strong enough evidence to reject the null hypothesis.

There could still be a real effect or difference, but it might be smaller or more variable than the study was able to detect.

Other factors like sample size, study design, and measurement precision can influence the p-value. It’s important to consider the entire body of evidence and not rely solely on p-values when interpreting research findings.

Can P values be exactly zero?

While a p-value can be extremely small, it cannot technically be absolute zero. When a p-value is reported as p = 0.000, the actual p-value is too small for the software to display. This is often interpreted as strong evidence against the null hypothesis. For p values less than 0.001, report as p < .001

Further Information

  • P Value Calculator From T Score
  • P-Value Calculator For Chi-Square
  • P-values and significance tests (Kahn Academy)
  • Hypothesis testing and p-values (Kahn Academy)
  • Wasserstein, R. L., Schirm, A. L., & Lazar, N. A. (2019). Moving to a world beyond “ p “< 0.05”.
  • Criticism of using the “ p “< 0.05”.
  • Publication manual of the American Psychological Association
  • Statistics for Psychology Book Download

Bland, J. M., & Altman, D. G. (1994). One and two sided tests of significance: Authors’ reply.  BMJ: British Medical Journal ,  309 (6958), 874.

Goodman, S. N., & Royall, R. (1988). Evidence and scientific research.  American Journal of Public Health ,  78 (12), 1568-1574.

Goodman, S. (2008, July). A dirty dozen: twelve p-value misconceptions . In  Seminars in hematology  (Vol. 45, No. 3, pp. 135-140). WB Saunders.

Lang, J. M., Rothman, K. J., & Cann, C. I. (1998). That confounded P-value.  Epidemiology (Cambridge, Mass.) ,  9 (1), 7-8.

Print Friendly, PDF & Email

  • Skip to secondary menu
  • Skip to main content
  • Skip to primary sidebar

Statistics By Jim

Making statistics intuitive

Failing to Reject the Null Hypothesis

By Jim Frost 69 Comments

Failing to reject the null hypothesis is an odd way to state that the results of your hypothesis test are not statistically significant. Why the peculiar phrasing? “Fail to reject” sounds like one of those double negatives that writing classes taught you to avoid. What does it mean exactly? There’s an excellent reason for the odd wording!

In this post, learn what it means when you fail to reject the null hypothesis and why that’s the correct wording. While accepting the null hypothesis sounds more straightforward, it is not statistically correct!

Before proceeding, let’s recap some necessary information. In all statistical hypothesis tests, you have the following two hypotheses:

  • The null hypothesis states that there is no effect or relationship between the variables.
  • The alternative hypothesis states the effect or relationship exists.

We assume that the null hypothesis is correct until we have enough evidence to suggest otherwise.

After you perform a hypothesis test, there are only two possible outcomes.

drawing of blind justice.

  • When your p-value is greater than your significance level, you fail to reject the null hypothesis. Your results are not significant. You’ll learn more about interpreting this outcome later in this post.

Related posts : Hypothesis Testing Overview and The Null Hypothesis

Why Don’t Statisticians Accept the Null Hypothesis?

To understand why we don’t accept the null, consider the fact that you can’t prove a negative. A lack of evidence only means that you haven’t proven that something exists. It does not prove that something doesn’t exist. It might exist, but your study missed it. That’s a huge difference and it is the reason for the convoluted wording. Let’s look at several analogies.

Species Presumed to be Extinct

Photograph of an Australian Tree Lobster.

Lack of proof doesn’t represent proof that something doesn’t exist!

Criminal Trials

Photograph of a gavel with law books.

Perhaps the prosecutor conducted a shoddy investigation and missed clues? Or, the defendant successfully covered his tracks? Consequently, the verdict in these cases is “not guilty.” That judgment doesn’t say the defendant is proven innocent, just that there wasn’t enough evidence to move the jury from the default assumption of innocence.

Hypothesis Tests

The Greek sympol of alpha, which represents the significance level.

The hypothesis test assesses the evidence in your sample. If your test fails to detect an effect, it’s not proof that the effect doesn’t exist. It just means your sample contained an insufficient amount of evidence to conclude that it exists. Like the species that were presumed extinct, or the prosecutor who missed clues, the effect might exist in the overall population but not in your particular sample. Consequently, the test results fail to reject the null hypothesis, which is analogous to a “not guilty” verdict in a trial. There just wasn’t enough evidence to move the hypothesis test from the default position that the null is true.

The critical point across these analogies is that a lack of evidence does not prove something does not exist—just that you didn’t find it in your specific investigation. Hence, you never accept the null hypothesis.

Related post : The Significance Level as an Evidentiary Standard

What Does Fail to Reject the Null Hypothesis Mean?

Accepting the null hypothesis would indicate that you’ve proven an effect doesn’t exist. As you’ve seen, that’s not the case at all. You can’t prove a negative! Instead, the strength of your evidence falls short of being able to reject the null. Consequently, we fail to reject it.

Failing to reject the null indicates that our sample did not provide sufficient evidence to conclude that the effect exists. However, at the same time, that lack of evidence doesn’t prove that the effect does not exist. Capturing all that information leads to the convoluted wording!

What are the possible implications of failing to reject the null hypothesis? Let’s work through them.

First, it is possible that the effect truly doesn’t exist in the population, which is why your hypothesis test didn’t detect it in the sample. Makes sense, right? While that is one possibility, it doesn’t end there.

Another possibility is that the effect exists in the population, but the test didn’t detect it for a variety of reasons. These reasons include the following:

  • The sample size was too small to detect the effect.
  • The variability in the data was too high. The effect exists, but the noise in your data swamped the signal (effect).
  • By chance, you collected a fluky sample. When dealing with random samples, chance always plays a role in the results. The luck of the draw might have caused your sample not to reflect an effect that exists in the population.

Notice how studies that collect a small amount of data or low-quality data are likely to miss an effect that exists? These studies had inadequate statistical power to detect the effect. We certainly don’t want to take results from low-quality studies as proof that something doesn’t exist!

However, failing to detect an effect does not necessarily mean a study is low-quality. Random chance in the sampling process can work against even the best research projects!

If you’re learning about hypothesis testing and like the approach I use in my blog, check out my eBook!

Cover image of my Hypothesis Testing: An Intuitive Guide ebook.

Share this:

is rejecting null hypothesis good

Reader Interactions

' src=

May 8, 2024 at 9:08 am

Thank you very much for explaining the topic. It brings clarity and makes statistics very simple and interesting. Its helping me in the field of Medical Research.

' src=

February 26, 2024 at 7:54 pm

Hi Jim, My question is that can I reverse Null hyposthesis and start with Null: µ1 ≠ µ2 ? Then, if I can reject Null, I will end up with µ1=µ2 for mean comparison and this what I am looking for. But isn’t this cheating?

' src=

February 26, 2024 at 11:41 pm

That can be done but it requires you to revamp the entire test. Keep in mind that the reason you normally start out with the null equating to no relationship is because the researchers typically want to prove that a relationship or effect exists. This format forces the researchers to collect a substantial amount of high quality data to have a chance at demonstrating that an effect exists. If they collect a small sample and/or poor quality (e.g., noisy or imprecise), then the results default back to the null stating that no effect exists. So, they have to collect good data and work hard to get findings that suggest the effect exists.

There are tests that flip it around as you suggest where the null states that a relationship does exist. For example, researchers perform an equivalency test when they want to show that there is no difference. That the groups are equal. The test is designed such that it requires a good sample size and high quality data to have a chance at proving equivalency. If they have a small sample size and/or poor quality data, the results default back to the groups being unequal, which is not what they want to show.

So, choose the null hypothesis and corresponding analysis based on what you hope to find. Choose the null hypothesis that forces you to work hard to reject it and get the results that you want. It forces you to collect better evidence to make your case and the results default back to what you don’t want if you do a poor job.

I hope that makes sense!

' src=

October 13, 2023 at 5:10 am

Really appreciate how you have been able to explain something difficult in very simple terms. Also covering why you can’t accept a null hypothesis – something which I think is frequently missed. Thank you, Jim.

' src=

February 22, 2022 at 11:18 am

Hi Jim, I really appreciate your blog, making difficult things sound simple is a great gift.

I have a doubt about the p-value. You said there are two options when it comes to hypothesis tests results . Reject or failing to reject the null, depending on the p-value and your significant level.

But… a P-value of 0,001 means a stronger evidence than a P-value of 0,01? ( both with a significant level of 5%. Or It doesn`t matter, and just every p-Value under your significant level means the same burden of evidence against the null?

I hope I made my point clear. Thanks a lot for your time.

February 23, 2022 at 9:06 pm

There are different schools of thought about this question. The traditional approach is clear cut. Your results are statistically significance when your p-value is less than or equal to your significance level. When the p-value is greater than the significance level, your results are not significant.

However, as you point out, lower p-values indicate stronger evidence against the null hypothesis. I write about this aspect of p-values in several articles, interpreting p-values (near the end) and p-values and reproducibility .

Personally, I consider both aspects. P-values near 0.05 provide weak evidence. Consequently, I’d be willing to say that p-values less than or equal to 0.05 are statistically significant, but when they’re near 0.05, I’d consider it as a preliminary result that requires more research. However, if the p-value is less 0.01, or even better 0.001, then that’s much stronger evidence and I’ll give those results more weight in my evaluation.

If you read those two articles, I think you’ll see what I mean.

' src=

January 1, 2022 at 6:00 pm

HI, I have a quick question that you may be able to help me with. I am using SPSS and carrying out a Mann W U Test it says to retain the null hypothesis. The hypothesis is that males are faster than women at completing a task. So is that saying that they are or are not

January 1, 2022 at 8:17 pm

In that case, your sample data provides insufficient evidence to conclude that males are faster. The results do not prove that males and females are the same speed. You just don’t have enough evidence to say males are faster. In this post, I cover the reasons why you can’t prove the null is true.

' src=

November 23, 2021 at 5:36 pm

What if I have to prove in my hypothesis that there shouldn’t be any affect of treatment on patients? Can I say that if my null hypothesis is accepted i have got my results (no effect)? I am confused what to do in this situation. As for null hypothesis we always have to write it with some type of equality. What if I want my result to be what i have stated in null hypothesis i.e. no effect? How to write statements in this case? I am using non parametric test, Mann whitney u test

November 27, 2021 at 4:56 pm

You need to perform an equivalence test, which is a special type of procedure when you want to prove that the results are equal. The problem with a regular hypothesis test is that when you fail to reject the null, you’re not proving that they the outcomes are equal. You can fail to reject the null thanks to a small sample size, noisy data, or a small effect size even when the outcomes are truly different at the population level. An equivalence test sets things up so you need strong evidence to really show that two outcomes are equal.

Unfortunately, I don’t have any content for equivalence testing at this point, but you can read an article about it at Wikipedia: Equivalence Test .

' src=

August 13, 2021 at 9:41 pm

Great explanation and great analogies! Thanks.

' src=

August 11, 2021 at 2:02 am

I got problems with analysis. I did wound healing experiments with drugs treatment (total 9 groups). When I do the 2-way ANOVA in excel, I got the significant results in sample (Drug Treatment) and columns (Day, Timeline) . But I did not get the significantly results in interactions. Can I still reject the null hypothesis and continue the post-hoc test?

Thank you very much.

' src=

June 13, 2021 at 4:51 am

Hi Jim, There are so many books covering maths/programming related to statistics/DS, but may be hardly any book to develop an intuitive understanding. Thanks to you for filling up that gap. After statistics, hypothesis-testing, regression, will it be possible for you to write such books on more topics in DS such as trees, deep-learning etc.

I recently started with reading your book on hypothesis testing (just finished the first chapter). I have a question w.r.t the fuel cost example (from first chapter), where a random sample of 25 families (with sample mean 330.6) is taken. To do the hypothesis testing here, we are taking a sampling distribution with a mean of 260. Then based on the p-value and significance level, we find whether to reject or accept the null hypothesis. The entire decision (to accept or reject the null hypothesis) is based on the sampling distribution about which i have the following questions : a) we are assuming that the sampling distribution is normally distributed. what if it has some other distribution, how can we find that ? b) We have assumed that the sampling distribution is normally distributed and then further assumed that its mean is 260 (as required for the hypothesis testing). But we need the standard deviation as well to define the normal distribution, can you please let me know how do we find the standard deviation for the sampling distribution ? Thanks.

' src=

April 24, 2021 at 2:25 pm

Maybe its the idea of “Innocent until proven guilty”? Your Null assume the person is not guilty, and your alternative assumes the person is guilty, only when you have enough evidence (finding statistical significance P0.05 you have failed to reject null hypothesis, null stands,implying the person is not guilty. Or, the person remain innocent.. Correct me if you think it’s wrong but this is the way I interpreted.

April 25, 2021 at 5:10 pm

I used the courtroom/trial analogy within this post. Read that for more details. I’d agree with your general take on the issue except when you have enough evidence you actually reject the null, which in the trial means the defendant is found guilty.

' src=

April 17, 2021 at 6:10 am

Can regression analysis be done using 5 companies variables for predicting working capital management and profitability positive/negative relationship?

Also, does null hypothesis rejecting means whatsoever is stated in null hypothesis that is false proved through regression analysis?

I have very less knowledge about regression analysis. Please help me, Sir. As I have my project report due on next week. Thanks in advance!

April 18, 2021 at 10:48 pm

Hi Ahmed, yes, regression analysis can be used for the scenario you describe as long as you have the required data.

For more about the null hypothesis in relation to regression analysis, read my post about regression coefficients and their p-values . I describe the null hypothesis in it.

' src=

January 26, 2021 at 7:32 pm

With regards to the legal example above. While your explanation makes sense when simplified to this statistical level, from a legal perspective it is not correct. The presumption of innocence means one does not need to be proven innocent. They are innocent. The onus of proof lies with proving they are guilty. So if you can’t prove someones guilt then in fact you must accept the null hypothesis that they are innocent. It’s not a statistical test so a little bit misleading using it an example, although I see why you would.

If it were a statistical test, then we would probably be rather paranoid that everyone is a murderer but they just haven’t been proven to be one yet.

Great article though, a nice simple and thoughtout explanation.

January 26, 2021 at 9:11 pm

It seems like you misread my post. The hypothesis testing/legal analogy is very strong both in making the case and in the result.

In hypothesis testing, the data have to show beyond a reasonable doubt that the alternative hypothesis is true. In a court case, the prosecutor has to present sufficient evidence to show beyond a reasonable doubt that the defendant is guilty.

In terms of the test/case results. When the evidence (data) is insufficient, you fail to reject the null hypothesis but you do not conclude that the data proves the null is true. In a legal case that has insufficient evidence, the jury finds the defendant to be “not guilty” but they do not say that s/he is proven innocent. To your point specifically, it is not accurate to say that “not guilty” is the same as “proven innocent.”

It’s a very strong parallel.

' src=

January 9, 2021 at 11:45 am

Just a question, in my research on hypotheses for an assignment, I am finding it difficult to find an exact definition for a hypothesis itself. I know the defintion, but I’m looking for a citable explanation, any ideas?

January 10, 2021 at 1:37 am

To be clear, do you need to come up with a statistical hypothesis? That’s one where you’ll use a particular statistical hypothesis test. If so, I’ll need to know more about what you’re studying, your variables, and the type of hypothesis test you plan to use.

There are also scientific hypotheses that you’ll state in your proposals, study papers, etc. Those are different from statistical hypotheses (although related). However, those are very study area specific and I don’t cover those types on this blog because this is a statistical blog. But, if it’s a statistical hypothesis for a hypothesis test, then let me know the information I mention above and I can help you out!

' src=

November 7, 2020 at 8:33 am

Hi, good read, I’m kind of a novice here, so I’m trying to write a research paper, and I’m trying to make a hypothesis. however looking at the literature, there are contradicting results.

researcher A found that there is relationship between X and Y

however, researcher B found that there is no relationship between X and Y

therefore, what is the null hypothesis between X and y? do we choose what we assumed to be correct for our study? or is is somehow related to the alternative hypothesis? I’m confused.

thank you very much for the help.

November 8, 2020 at 12:07 am

Hypotheses for a statistical test are different than a researcher’s hypothesis. When you’re constructing the statistical hypothesis, you don’t need to consider what other researchers have found. Instead, you construct them so that the test only produces statistically significant results (rejecting the null) when your data provides strong evidence. I talk about that process in this post.

Typically, researchers are hoping to establish that an effect or relationship exists. Consequently, the null and alternative hypotheses are typically the following:

Null: The effect or relationship doesn’t not exist. Alternative: The effect or relationship does exist.

However, if you’re hoping to prove that there is no effect or no relationship, you then need to flip those hypotheses and use a special test, such as an equivalences test.

So, there’s no need to consider what researchers have found but instead what you’re looking for. In most cases, you are looking for an effect/relationship, so you’d go with the hypotheses as I show them above.

I hope that helps!

' src=

October 22, 2020 at 6:13 pm

Great, deep detailed answer. Appreciated!

' src=

September 16, 2020 at 12:03 pm

Thank you for explaining it too clearly. I have the following situation with a Box Bohnken design of three levels and three factors for multiple responses. F-value for second order model is not significant (failing to reject null hypothesis, p-value > 0.05) but, lack of fit of the model is not significant. What can you suggest me about statistical analysis?

September 17, 2020 at 2:42 am

Are your first order effects significant?

You want the lack of fit to be nonsignificant. If it’s significant, that means the model doesn’t fit the data well. So, you’re good there! 🙂

' src=

September 14, 2020 at 5:18 pm

thank you for all the explicit explanation on the subject.

However, i still got a question about “accepting the null hypothesis”. from textbook, the p-value is the probability that a statistic would take a value that is as extreme as or more extreme than that actually observed.

so, that’s why when p<0.01 we reject the null hypothesis, because it's too rare (p0.05, i can understand that for most cases we cannot accept the null, for example, if p=0.5, it means that the probability to get a statistic from the distribution is 0.5, which is totally random.

But how about when the p is very close to 1, like p=0.95, or p=0.99999999, can’t we say that the probability that the statistic is not from this distribution is less than 0.05, | or in another way, the probability that the statistic is from the distribution is almost 1. can’t we accept the null in such circumstance?

' src=

September 11, 2020 at 12:14 pm

Wow! This is beautifully explained. “Lack of proof doesn’t represent proof that something doesn’t exist!”. This kinda, hit me with such force. Can I then, use the same analogy for many other things in life? LOL! 🙂

H0 = God does not exist; H1 = God does exist; WE fail to reject H0 as there is no evidence.

Thank you sir, this has answered many of my questions, statistically speaking! No pun intended with the above.

September 11, 2020 at 4:58 pm

Hi, LOL, I’m glad it had such meaning for you! I’ll leave the determination about the existence of god up to each person, but in general, yes, I think statistical thinking can be helpful when applied to real life. It is important to realize that lack of proof truly is not proof that something doesn’t exist. But, I also consider other statistical concepts, such as confounders and sampling methodology, to be useful keeping in mind when I’m considering everyday life stuff–even when I’m not statistically analyzing it. Those concepts are generally helpful when trying to figure out what is going on in your life! Are there other alternative explanations? Is what you’re perceiving likely to be biased by something that’s affecting the “data” you can observe? Am I drawing a conclusion based on a large or small sample? How strong is the evidence?

A lot of those concepts are great considerations even when you’re just informally assessing and draw conclusions about things happening in your daily life.

' src=

August 13, 2020 at 12:04 am

Dear Jim, thanks for clarifying. absolutely, now it makes sense. the topic is murky but it is good to have your guidance, and be clear. I have not come across an instructor as clear in explaining as you do. Appreciate your direction. Thanks a lot, Geetanjali

August 15, 2020 at 3:48 pm

Hi Geetanjali,

I’m glad my website is helpful! That makes my day hearing that. Thanks so much for writing!

' src=

August 12, 2020 at 9:37 am

Hi Jim. I am doing data analyis for my masters thesis and my hypothesis testings were insignificant. And I am ok with that. But there is something bothering me. It is the low reliabilities of the 4-Items sub-scales (.55, .68, .75), though the overall alpha is good (.85). I just wonder if it is affecting my hypothesis testings.

' src=

August 11, 2020 at 9:23 pm

Thank you sir for replying, yes sir we it’s a RCT study.. where we did within and between the groups analysis and found p>0.05 in between the groups using Mann Whitney U test. So in such cases if the results comes like this we need to Mention that we failed reject the null hypothesis? Is that correct? Whether it tells that the study is inefficient as we couldn’t accept the alternative hypothesis. Thanks is advance.

August 11, 2020 at 9:43 pm

Hi Saumya, ah, this becomes clearer. When ask statistical questions, please be sure to include all relevant information because the details are extremely important. I didn’t know it was an RCT with a treatment and control group. Yes, given that your p-value is greater than your significance level, you fail to reject the null hypothesis. The results are not significant. The experiment provides insufficient evidence to conclude that the outcome in the treatment group is different than the control group.

By the way, you never accept the alternative hypothesis (or the null). The two options are to either reject the null or fail to reject the null. In your case, you fail to reject the null hypothesis.

I hope this helps!

August 11, 2020 at 9:41 am

Sir, p value is0.05, by which we interpret that both the groups are equally effective. In this case I had to reject the alternative hypothesis/ failed to reject null hypothessis.

August 11, 2020 at 12:37 am

sir, within the group analysis the p value for both the groups is significant (p0.05, by which we interpret that though both the treatments are effective, there in no difference between the efficacy of one over the other.. in other words.. no intervention is superior and both are equally effective.

August 11, 2020 at 2:45 pm

Thanks for the additional details. If I understand correctly, there were separate analyses before that determined each treatment had a statistically significance effect. However, when you compare the two treatments, there difference between them is not statistically significant.

If that’s the case, the interpretation is fairly straightforward. You have evidence that suggests that both treatments are effective. However, you don’t have evidence to conclude that one is better than the other.

August 10, 2020 at 9:26 am

Hi thank you for a wonderful explanation. I have a doubt: My Null hypothesis says: no significant difference between the effect fo A and B treatment Alternative hypothesis: there will be significant difference between the effect of A and B treatment. and my results show that i fail to reject null hypothesis.. Both the treatments were effective, but not significant difference.. how do I interpret this?

August 10, 2020 at 1:32 pm

First, I need to ask you a question. If your p-value is not significant, and so you fail to reject the null, why do you say that the treatment is effective? I can answer you question better after knowing the reason you say that. Thanks!

August 9, 2020 at 9:40 am

Dear Jim, thanks for making stats much more understandable and answering all question so painstakingly. I understand the following on p value and null. If our sample yields a p value of .01, it means that that there is a 1% probability that our kind of sample exists in the population. that is a rare event. So why shouldn’t we accept the HO as the probability of our event was v rare. Pls can you correct me. Thanks, G

August 10, 2020 at 1:53 pm

That’s a great question! They key thing to remember is that p-values are a conditional probability. P-value calculations assume that the null hypothesis is true. So, a p-value of 0.01 indicates that there is a 1% probability of observing your sample results, or more extreme, *IF* the null hypothesis is true.

The kicker is that we don’t whether the null is true or not. But, using this process does limit the likelihood of a false positive to your significance level (alpha). But, we don’t know whether the null is true and you had an unusual sample or whether the null is false. Usually, with a p-value of 0.01, we’d reject the null and conclude it is false.

I hope that answered your question. This topic can be murky and I wasn’t quite clear which part you needed clarification.

' src=

August 4, 2020 at 11:16 pm

Thank you for the wonderful explanation. However, I was just curious to know that what if in a particular test, we get a p-value less than the level of significance, leading to evidence against null hypothesis. Is there any possibility that our interpretation of population effect might be wrong due to randomness of samples? Also, how do we conclude whether the evidence is enough for our alternate hypothesis?

August 4, 2020 at 11:55 pm

Hi Abhilash,

Yes, unfortunately, when you’re working with samples, there’s always the possibility that random chance will cause your sample to not represent the population. For information about these errors, read my post about the types of errors in hypothesis testing .

In hypothesis testing, you determine whether your evidence is strong enough to reject the null. You don’t accept the alternative hypothesis. I cover that in my post about interpreting p-values .

' src=

August 1, 2020 at 3:50 pm

Hi, I am trying to interpret this phenomenon after my research. The null hypothesis states that “The use of combined drugs A and B does not lower blood pressure when compared to if drug A or B is used singularly”

The alternate hypothesis states: The use of combined drugs A and B lower blood pressure compared to if drug A or B is used singularly.

At the end of the study, majority of the people did not actually combine drugs A and B, rather indicated they either used drug A or drug B but not a combination. I am finding it very difficult to explain this outcome more so that it is a descriptive research. Please how do I go about this? Thanks a lot

' src=

June 22, 2020 at 10:01 am

What confuses me is how we set/determine the null hypothesis? For example stating that two sets of data are either no different or have no relationship will give completely different outcomes, so which is correct? Is the null that they are different or the same?

June 22, 2020 at 2:16 pm

Typically, the null states there is no effect/no relationship. That’s true for 99% of hypothesis tests. However, there are some equivalence tests where you are trying to prove that the groups are equal. In that case, the null hypothesis states that groups are not equal.

The null hypothesis is typically what you *don’t* want to find. You have to work hard, design a good experiment, collect good data, and end up with sufficient evidence to favor the alternative hypothesis. Usually in an experiment you want to find an effect. So, usually the null states there is no effect and you have get good evidence to reject that notion.

However, there are a few tests where you actually want to prove something is equal, so you need the null to state that they’re not equal in those cases and then do all the hard work and gather good data to suggest that they are equal. Basically, set up the hypothesis so it takes a good experiment and solid evidence to be able to reject the null and favor the hypothesis that you’re hoping is true.

' src=

June 5, 2020 at 11:54 am

Thank you for the explanation. I have one question that. If Null hypothesis is failed to reject than is possible to interpret the analysis further?

June 5, 2020 at 7:36 pm

Hi Mottakin,

Typically, if your result is that you fail to reject the null hypothesis there’s not much further interpretation. You don’t want to be in a situation where you’re endlessly trying new things on a quest for obtaining significant results. That’s data mining.

' src=

May 25, 2020 at 7:55 am

I hope all is well. I am enjoying your blog. I am not a statistician, however, I use statistical formulae to provide insight on the direction in which data is going. I have used both the regression analysis and a T-Test. I know that both use a null hypothesis and an alternative hypothesis. Could you please clarity the difference between a regression analysis and a T-Test? Are there conditions where one is a better option than the other?

May 26, 2020 at 9:18 pm

t-Tests compare the means of one or two groups. Regression analysis typically describes the relationships between a set of independent variables and the dependent variables. Interestingly, you can actually use regression analysis to perform a t-test. However, that would be overkill. If you just want to compare the means of one or two groups, use a t-test. Read my post about performing t-tests in Excel to see what they can do. If you have a more complex model than just comparing one or two means, regression might be the way to go. Read my post about when to use regression analysis .

' src=

May 12, 2020 at 5:45 pm

This article is really enlightening but there is still some darkness looming around. I see that low p-values mean strong evidence against null hypothesis and finding such a sample is highly unlikely when null hypothesis is true. So , is it OK to say that when p-value is 0.01 , it was very unlikely to have found such a sample but we still found it and hence finding such a sample has not occurred just by chance which leads towards rejection of null hypothesis.

May 12, 2020 at 11:16 pm

That’s mostly correct. I wouldn’t say, “has not occurred by chance.” So, when you get a very low p-value it does mean that you are unlikely to obtain that sample if the null is true. However, once you obtain that result, you don’t know for sure which of the two occurred:

  • The effect exists in the population.
  • Random chance gave you an unusual sample (i.e., Type I error).

You really don’t know for sure. However, by the decision making results you set about the strength of evidence required to reject the null, you conclude that the effect exists. Just always be aware that it could be a false positive.

That’s all a long way of saying that your sample was unlikely to occur by chance if the null is true.

' src=

April 29, 2020 at 11:59 am

Why do we consult the statistical tables to find out the critical values of our test statistics?

April 30, 2020 at 5:05 pm

Statistical tables started back in the “olden days” when computers didn’t exist. You’d calculate the test statistic value for your sample. Then, you’d look in the appropriate table and using the degrees of freedom for your design and find the critical values for the test statistic. If the value of your test statistics exceeded the critical value, your results were statistically significant.

With powerful and readily available computers, researchers could analyze their data and calculate the p-values and compare them directly to the significance level.

I hope that answers your question!

' src=

April 15, 2020 at 10:12 am

If we are not able to reject the null hypothesis. What could be the solution?

April 16, 2020 at 11:13 pm

Hi Shazzad,

The first thing to recognize is that failing to reject the null hypothesis might not be an error. If the null hypothesis is false, then the correct outcome is failing to reject the null.

However, if the null hypothesis is false and you fail to reject, it is a type II error, or a false negative. Read my post about types of errors in hypothesis tests for more information.

This type of error can occur for a variety of reasons, including the following:

  • Fluky sample. When working with random samples, random error can cause anomalous results purely by chance.
  • Sample is too small. Perhaps the sample was too small, which means the test didn’t have enough statistical power to detect the difference.
  • Problematic data or sampling methodology. There could be a problem with how you collected the data or your sampling methodology.

There are various other possibilities, but those are several common problems.

' src=

April 14, 2020 at 12:19 pm

Thank you so much for this article! I am taking my first Statistics class in college and I have one question about this.

I understand that the default position is that the null is correct, and you explained that (just like a court case), the sample evidence must EXCEED the “evidentiary standard” (which is the significance level) to conclude that an effect/relationship exists. And, if an effect/relationship exists, that means that it’s the alternative hypothesis that “wins” (not sure if that’s the correct way of wording it, but I’m trying to make this as simple as possible in my head!).

But what I don’t understand is that if the P-value is GREATER than the significance value, we fail to reject the null….because shouldn’t a higher P-value, mean that our sample evidence EXCEEDS the evidentiary standard (aka the significance level), and therefore an effect/relationship exists? In my mind it would make more sense to reject the null, because our P-value is higher and therefore we have enough evidence to reject the null.

I hope I worded this in a way that makes sense. Thank you in advance!

April 14, 2020 at 10:42 pm

That’s a great question. The key thing to remember is that higher p-values correspond to weaker evidence against the null hypothesis. A high p-value indicates that your sample is likely (high probability = high p-value) if the null hypothesis is true. Conversely, low p-values represent stronger evidence against the null. You were unlikely (low probability = low p-value) to have collect a sample with the measured characteristics if the null is true.

So, there is negative correlation between p-values and strength of evidence against the null hypothesis. Low p-values indicate stronger evidence. Higher p-value represent weaker evidence.

In a nutshell, you reject the null hypothesis with a low p-value because it indicates your sample data are unusual if the null is true. When it’s unusual enough, you reject the null.

' src=

March 5, 2020 at 11:10 am

There is something I am confused about. If our significance level is .05 and our resulting p-value is .02 (thus the strength of our evidence is strong enough to reject the null hypothesis), do we state that we reject the null hypothesis with 95% confidence or 98% confidence?

My guess is our confidence level is 95% since or alpha was .05. But if the strength of our evidence is 98%, why wouldn’t we use that as our stated confidence in our results?

March 5, 2020 at 4:19 pm

Hi Michael,

You’d state that you can reject the null at a significance level of 5% or conversely at the 95% confidence level. A key reason is to avoid cherry picking your results. In other words, you don’t want to choose the significance level based on your results.

Consequently, set the significance level/confidence level before performing your analysis. Then, use those preset levels to determine statistical significance. I always recommend including the exact p-value when you report on statistical significance. Exact p-values do provide information about the strength of evidence against the null.

' src=

March 5, 2020 at 9:58 am

Thank you for sharing this knowledge , it is very appropriate in explaining some observations in the study of forest biodiversity.

' src=

March 4, 2020 at 2:01 am

Thank you so much. This provides for my research

' src=

March 3, 2020 at 7:28 pm

If one couples this with what they call estimated monetary value of risk in risk management, one can take better decisions.

' src=

March 3, 2020 at 3:12 pm

Thank you for providing this clear insight.

March 3, 2020 at 3:29 am

Nice article Jim. The risk of such failure obviously reduces when a lower significance level is specified.One benefits most by reading this article in conjunction with your other article “Understanding Significance Levels in Statistics”.

' src=

March 3, 2020 at 2:43 am

That’s fine. My question is why doesn’t the numerical value of type 1 error coincide with the significance level in the backdrop that the type 1 error and the significance level are both the same ? I hope you got my question.

March 3, 2020 at 3:30 am

Hi, they are equal. As I indicated, the significance level equals the type I error rate.

March 3, 2020 at 1:27 am

Kindly elighten me on one confusion. We set out our significance level before setting our hypothesis. When we calculate the type 1 error, which happens to be a significance level, the numerical value doesn’t equals (either undermining value comes out or an exceeding value comescout ) our significance level that was preassigned. Why is this so ?

March 3, 2020 at 2:24 am

Hi Ratnadeep,

You’re correct. The significance level (alpha) is the same as the type I error rate. However, you compare the p-value to the significance level. It’s the p-value that can be greater than or less than the significance level.

The significance level is the evidentiary standard. How strong does the evidence in your sample need to be before you can reject the null? The p-value indicates the strength of the evidence that is present in your sample. By comparing the p-value to the significance level, you’re comparing the actual strength of the sample evidence to the evidentiary standard to determine whether your sample evidence is strong enough to conclude that the effect exists in the population.

I write about this in my post about the understanding significance levels . I think that will help answer your questions!

Comments and Questions Cancel reply

Stack Exchange Network

Stack Exchange network consists of 183 Q&A communities including Stack Overflow , the largest, most trusted online community for developers to learn, share their knowledge, and build their careers.

Q&A for work

Connect and share knowledge within a single location that is structured and easy to search.

On the logic meaning behind rejecting the null hypothesis and its relantionship with Type I error

I understand that the p-value is defined as the probability to obtain a "more extreme" value of $w$ if $H_0$ is true, i.e.

$p=P(|W| > |w| \ |H_0 $ is true $)$

and the "significance level" $\alpha$ is a threshold to decide if it is "admissible" that $w$ comes in null hypothesis setting, i.e.

$p < \alpha \Rightarrow$ reject $H_0$ .

However, It is hard for me to understand the meaning of hypothesis test when $\alpha$ is interpreted as a probability, specifically:

$\alpha=P($ reject $H_0 | H_0 $ is true $)$ .

In this case, I will have:

$P(|W| > |w| \ |H_0 $ is true $) \ < P($ reject $H_0 | H_0 $ is true $)\Rightarrow$ reject $H_0$

Therefore I'm saying that, if $H_0$ is true, the probability of $w$ is less than the probability of rejecting $H_0$ , therefore reject $H_0$ . Since, for example, I obtained $w$ , its probability is lower than the probability of in rejecting $H_0$ , so reject. In other words, it is more probable that I reject when I shouldn't, respect to the fact that $w$ has been sampled, so reject. This seems to me that it is more probable that I'm making a reject when I shouldn't, or there is something that escapes me. Furthermore, I'm comparing two different distributions, i.e. $P(W)$ and $P(\text{ reject } H_0)$ , that seems to me two different objects, therefore I don't understand the point of comparing them.

This question can be viewed similar to The rationale behind the "fail to reject the null" jargon in hypothesis testing? , but in my opinion is different (it regards also the type II error, which is not the subject of my question) and also to What is the meaning of p values and t values in statistical tests? (which see $\alpha$ simply as a threshold, and not as a probability): Maybe a similiar question is intuition/logic behind comparing p-value and significance level but the given answer is not fully satisfactory for me, and the question has been closed since considered too similar to the other questions.

So, what is the logical meaning in rejecting the null-hypothesis when $\alpha$ is viewed as the probability to make a Type I mistake?

  • hypothesis-testing
  • statistical-significance
  • type-i-and-ii-errors

User1865345's user avatar

Until you study Bayesian inference this will not make as much sense to you. But you started off incorrectly. With continuous data the probability of achieving any one value of a test statistics (or of a summary measure the test statistic is based on) is zero. So the p-value is the probability of getting a test statistic that is more extreme than the observed one if $H_0$ is true and the model is correct. To read more about the enormous difference between $\alpha$ and decision errors, read this .

$\alpha$ isn't the probability of making a mistake. This is one of the most common interpretation errors and is at the root of a lot of unclear thinking by a lot of practitioners. The term "type I error rate" is a misnomer that started us off on the wrong foot a century ago. It's not a rate and is not an error probability. That's why I've moved to the term "type I assertion probability $\alpha$ in its stead. It's just an assertion trigger probability. No conditional probability that assumes $H_0$ is true can inform you about the veracity of $H_0$ .

With Bayesian posterior probabilities you compute probabilities of any assertion you have and use direct reasoning. If you decide to act as if an effect is positive when P(effect > 0 | data, prior) = 0.98 you automatically get a decision error probability to carry along, which is simply 0.02.

Frank Harrell's user avatar

  • 2 $\begingroup$ I'm not a Bayesian and have only studied it a little, but your answer makes good sense to me. Maybe the little that I've studied it is enough. $\endgroup$ –  Peter Flom Commented Aug 16, 2023 at 11:34
  • 1 $\begingroup$ It's generally stated as $\Pr(W > w)$ but you can do it either way if you want a one-sided test. But most tests are two-sided so use something like $\Pr(|W| > |w|)$. $\endgroup$ –  Frank Harrell Commented Aug 16, 2023 at 12:21
  • 1 $\begingroup$ thank you. I tried to correct my question accordly, but still there is something missing in all the reasoning. More precisely, I don't understand what you mean with "assertion probability" $\endgroup$ –  volperossa Commented Aug 16, 2023 at 12:56
  • 2 $\begingroup$ In simplest terms one triggers an assertion of an effect (rejection of the supposition of no effect) when for example p < 0.05. The probability of making such an assertion under $H_0$ is 0.05 if p-values are accurately computed and the data model holds. $\endgroup$ –  Frank Harrell Commented Aug 16, 2023 at 14:00
  • 3 $\begingroup$ I have a semantic issue here. If in mathematical symbols we write $$\alpha=P({\rm reject }\, H_0 | H_0\, {\rm is\, true})$$, then is it not grammatically correct to translate this in words as "the probability of rejecting $H_0$ given that $H_0$ is true", which is an example of "making a mistake"? $\endgroup$ –  Alecos Papadopoulos Commented Aug 17, 2023 at 1:25

Your Answer

Sign up or log in, post as a guest.

Required, but never shown

By clicking “Post Your Answer”, you agree to our terms of service and acknowledge you have read our privacy policy .

Not the answer you're looking for? Browse other questions tagged hypothesis-testing statistical-significance p-value type-i-and-ii-errors or ask your own question .

  • Featured on Meta
  • Preventing unauthorized automated access to the network
  • User activation: Learnings and opportunities
  • Join Stack Overflow’s CEO and me for the first Stack IRL Community Event in...

Hot Network Questions

  • The answer is not ___
  • Do all languages distinguish between persons and non-persons?
  • In the absence of an agreement addressing the issue, is there any law giving a university copyright in an undergraduate student's class paper?
  • I want to find a smooth section of the map from the Stiefel manifold to the Grassmanian manifold
  • Is there a way to have my iPhone register my car which doesn't have carplay, only for the "Car is parked at"-feature?
  • Creating three versions of Beamer presentation
  • What is the simplest formula for calculating the circumference of a circle?
  • How important exactly is the Base Attack Bonus?
  • Are there individual protons and neutrons in a nucleus?
  • Complexity of computing minimum unsatisfiable core
  • How can moving observer explain non-simultaneity?
  • Is it even possible to build a beacon to announce we exist?
  • Is the Earth still capable of massive volcanism, like the kind that caused the formation of the Siberian Traps?
  • Musicians wearing Headphones
  • Secure flag cookies are sent on a non-secure connection
  • If a professor wants to hire a student themselves, how can they write a letter of recommendation for other universities?
  • How can the doctor measure out a dose (dissolved in water) of exactly 10% of a tablet?
  • Does Dragon Ball Z: Kakarot have ecchi scenes?
  • Pulling myself up with a pulley attached to myself
  • Can we divide the story points across team members?
  • "Chrisma" and "Him"
  • Rogue's Uncanny Dodge (reaction) versus Evasion on Dex saving throw
  • Soldering a thermal fuse. 92°C
  • CH in non-set theoretic foundations

is rejecting null hypothesis good

Null Hypothesis Examples

ThoughtCo / Hilary Allison

  • Scientific Method
  • Chemical Laws
  • Periodic Table
  • Projects & Experiments
  • Biochemistry
  • Physical Chemistry
  • Medical Chemistry
  • Chemistry In Everyday Life
  • Famous Chemists
  • Activities for Kids
  • Abbreviations & Acronyms
  • Weather & Climate
  • Ph.D., Biomedical Sciences, University of Tennessee at Knoxville
  • B.A., Physics and Mathematics, Hastings College

In statistical analysis, the null hypothesis assumes there is no meaningful relationship between two variables. Testing the null hypothesis can tell you whether your results are due to the effect of manipulating ​a dependent variable or due to chance. It's often used in conjunction with an alternative hypothesis, which assumes there is, in fact, a relationship between two variables.

The null hypothesis is among the easiest hypothesis to test using statistical analysis, making it perhaps the most valuable hypothesis for the scientific method. By evaluating a null hypothesis in addition to another hypothesis, researchers can support their conclusions with a higher level of confidence. Below are examples of how you might formulate a null hypothesis to fit certain questions.

What Is the Null Hypothesis?

The null hypothesis states there is no relationship between the measured phenomenon (the dependent variable ) and the independent variable , which is the variable an experimenter typically controls or changes. You do not​ need to believe that the null hypothesis is true to test it. On the contrary, you will likely suspect there is a relationship between a set of variables. One way to prove that this is the case is to reject the null hypothesis. Rejecting a hypothesis does not mean an experiment was "bad" or that it didn't produce results. In fact, it is often one of the first steps toward further inquiry.

To distinguish it from other hypotheses , the null hypothesis is written as ​ H 0  (which is read as “H-nought,” "H-null," or "H-zero"). A significance test is used to determine the likelihood that the results supporting the null hypothesis are not due to chance. A confidence level of 95% or 99% is common. Keep in mind, even if the confidence level is high, there is still a small chance the null hypothesis is not true, perhaps because the experimenter did not account for a critical factor or because of chance. This is one reason why it's important to repeat experiments.

Examples of the Null Hypothesis

To write a null hypothesis, first start by asking a question. Rephrase that question in a form that assumes no relationship between the variables. In other words, assume a treatment has no effect. Write your hypothesis in a way that reflects this.

Are teens better at math than adults? Age has no effect on mathematical ability.
Does taking aspirin every day reduce the chance of having a heart attack? Taking aspirin daily does not affect heart attack risk.
Do teens use cell phones to access the internet more than adults? Age has no effect on how cell phones are used for internet access.
Do cats care about the color of their food? Cats express no food preference based on color.
Does chewing willow bark relieve pain? There is no difference in pain relief after chewing willow bark versus taking a placebo.

Other Types of Hypotheses

In addition to the null hypothesis, the alternative hypothesis is also a staple in traditional significance tests . It's essentially the opposite of the null hypothesis because it assumes the claim in question is true. For the first item in the table above, for example, an alternative hypothesis might be "Age does have an effect on mathematical ability."

Key Takeaways

  • In hypothesis testing, the null hypothesis assumes no relationship between two variables, providing a baseline for statistical analysis.
  • Rejecting the null hypothesis suggests there is evidence of a relationship between variables.
  • By formulating a null hypothesis, researchers can systematically test assumptions and draw more reliable conclusions from their experiments.
  • What Are Examples of a Hypothesis?
  • Random Error vs. Systematic Error
  • Six Steps of the Scientific Method
  • What Is a Hypothesis? (Science)
  • Scientific Method Flow Chart
  • What Are the Elements of a Good Hypothesis?
  • Scientific Method Vocabulary Terms
  • Understanding Simple vs Controlled Experiments
  • The Role of a Controlled Variable in an Experiment
  • What Is an Experimental Constant?
  • What Is a Testable Hypothesis?
  • Scientific Hypothesis Examples
  • What Is the Difference Between a Control Variable and Control Group?
  • DRY MIX Experiment Variables Acronym
  • What Is a Controlled Experiment?
  • Scientific Variable

Gabriel Young Ph.D.

In Defense of Trying: Why Quitting and Failure are Valuable

Blind commitment & fear of falling short prevent us from living our best lives..

Updated September 23, 2024 | Reviewed by Ekua Hagan

  • What Is Resilience?
  • Take our Resilience Test
  • Find a therapist near me
  • People are given the message from an early age that they must succeed at everything they attempt.
  • Society encourages irrational commitment that squanders life's finite time.
  • The key to development is giving both children and adults permission to sample the many experiences of life.

Anastasia Shuraeva/Pexels

Trying gets a bad rap. I encourage my clients to try new things but I'm surprised at the level of resistance that commonly comes up. People often exhibit a strong and visceral aversion to my suggestion. When we drill down into what's going on, two themes emerge: fear of failing and fear of quitting. If we let these fears win out, we'll lead an empty life devoid of experience. But we can free ourselves from this trap by shifting how we think about trying.

Failure is good

The prevailing social judgment against failure is summed up in the quote from Yoda during The Empire Strikes Back in which the Jedi Master tells Luke, “Try not. Do or do not. There is no try” (Lucasfilm Ltd., n.d.). Not only am I unable to see the wisdom in these words, but they carry a connotation that I find counterproductive, or even downright harmful: that failure is not okay.

Failure is not just okay, it’s necessary for our development. For one thing, history demonstrates that people who attempt the monumental are met with failure time and again. Thomas Edison, Sojourner Truth, Gandhi, and many others have offered inspiring accounts of the role of defeat and failure in their developmental journeys. In our own time, President Obama, for example, has discussed how he was shaped by the difficulty he experienced as a young man both finding and keeping a job (e.g. Kovaleski, 2008). Michael Jordan stated, “I've missed more than 9,000 shots in my career . I've lost almost 300 games. Twenty-six times I've been trusted to take the game-winning shot and missed. I've failed over, and over and over again in my life. And that is why I succeed” (Zorn, 2018).

The stories of these and other influential people show us that failure is not only valuable as the means to the end of success, as Michael Jordan points out, but often failure is valuable in and of itself. It is an opportunity for learning, growth, and finding meaning in our struggles. One of the best fictionalized accounts of this truth can be found in the movie Rocky .

Many people forget that in the original film, Rocky loses at the end. We forget that because his failure feels like a victory. His failure demonstrated to him who he truly was and solidified the value of the relationships in his life. The fact that he tried was what defined him. We should all be so fortunate to experience such a loss.

But it’s not just the monumental loss that intimidates us. I’ve come to see that often times our fear of failure pushes us away from attempts even at the mundane. While going through graduate school, one of the ways I supported myself was by substitute teaching in grades K-12. I remember I was teaching a first-grade class around Thanksgiving and the lesson called for the class to make turkeys out of construction paper by tracing their hands. Even after a demonstration and repeated encouragement, the class just sat there because no child wanted to be the first to try. Even at this young age, they internalized the message that not trying was preferable to failure.

Quitting is Just Being Frugal with our Time

The Roman philosopher Seneca (n.d.) wrote, “It is not that we have so little time but that we lose so much... The life we receive is not short but we make it so; we are not ill provided but use what we have wastefully.” In my own life, I can think of many instances of people valuing irrational commitment but one example from junior high stands out. A teacher at my school berated me for discontinuing band. I had tried it, and not stuck with it, which she said, reflected poorly on my character. She called me a quitter. She didn’t take into consideration that my single mom could no longer afford the instrument rental, or that band practices were before school when I had to deliver newspapers in order to make ends meet. From my teacher’s privileged vantage point, merely trying band was a moral failure on my part. For me, I am grateful that I had the opportunity to experience it even for a while.

The Freedom of Rejecting "Growth Mindset"

In my years of teaching graduate school and practicing therapy , I’ve seen the impact of this destructive message over and over. No only do clients avoid taking up a new hobby because they feel they have to commit to it long-term as well as excel, but students are gripped with anxiety at small group exercises unless they are told exactly what to say and do. We’ve been conditioned that missing out on life experiences is preferable to looking foolish or being perceived as a quitter. These are some of the ways in which the growth mindset theory perpetuates maladaptive traits. We should feel emboldened to try and at the same time empowered to let go. We can’t ever find ourselves without permission to search in every nook and cranny life has to offer.

Kovaleski, S. (2008, July 7). Obama's organizing Years, Guiding others and finding himself. The New York Times. Retrieved September 13, 2021, from https://www.nytimes.com/2008/07/07/us/politics/07community.html .

Lucasfilm Ltd. (n.d.). Do. or do not. - star wars: The empire strikes back. StarWars.com. Retrieved September 13, 2021, from https://www.starwars.com/video/do-or-do-not .

Seneca, L. (n.d.). Seneca: On the shortness of life . Seneca: On the Shortness of Life | Online Philosophy. https://onlinephilosophy.org/texts/seneca-shortness-life

Zorn, E. (2018, August 30). WITHOUT failure, Jordan would be false idol. chicagotribune.com. Retrieved September 13, 2021, from https://webcache.googleusercontent.com/search?q=cache%3AJgMEuomTRtAJ%3A… .

Gabriel Young Ph.D.

Gabriel Young, Ph.D., is a licensed Marriage and Family Therapist, holding a Master's degree in Counseling Psychology and a Ph.D. in Human Development.

  • Find a Therapist
  • Find a Treatment Center
  • Find a Psychiatrist
  • Find a Support Group
  • Find Online Therapy
  • United States
  • Brooklyn, NY
  • Chicago, IL
  • Houston, TX
  • Los Angeles, CA
  • New York, NY
  • Portland, OR
  • San Diego, CA
  • San Francisco, CA
  • Seattle, WA
  • Washington, DC
  • Asperger's
  • Bipolar Disorder
  • Chronic Pain
  • Eating Disorders
  • Passive Aggression
  • Personality
  • Goal Setting
  • Positive Psychology
  • Stopping Smoking
  • Low Sexual Desire
  • Relationships
  • Child Development
  • Self Tests NEW
  • Therapy Center
  • Diagnosis Dictionary
  • Types of Therapy

September 2024 magazine cover

It’s increasingly common for someone to be diagnosed with a condition such as ADHD or autism as an adult. A diagnosis often brings relief, but it can also come with as many questions as answers.

  • Emotional Intelligence
  • Gaslighting
  • Affective Forecasting
  • Neuroscience

IMAGES

  1. Significance Level and Power of a Hypothesis Test Tutorial

    is rejecting null hypothesis good

  2. PPT

    is rejecting null hypothesis good

  3. Probability Of Rejecting The Null Hypothesis

    is rejecting null hypothesis good

  4. PPT

    is rejecting null hypothesis good

  5. 15 Null Hypothesis Examples (2024)

    is rejecting null hypothesis good

  6. The probability of rejecting the null hypothesis when, in fact, the

    is rejecting null hypothesis good

VIDEO

  1. Hypothesis Testing: the null and alternative hypotheses

  2. Easy understanding of null hypothesis part 2 #null hypothesis

  3. Zscore, null and alternate hypothesis testing

  4. The Concept of Rejecting a Null Hypothesis and the Steps of a Formal Hypothesis Test

  5. Methods of rejecting Null Hypothesis || Quantitative Methods || CFA Level-1

  6. Evidence to Reject the Null

COMMENTS

  1. What Is The Null Hypothesis & When To Reject It

    Rejecting the null hypothesis sets the stage for further experimentation to see if a relationship between two variables exists. Hypothesis testing is a critical part of the scientific method as it helps decide whether the results of a research study support a particular theory about a given population. Hypothesis testing is a systematic way of ...

  2. When Do You Reject the Null Hypothesis? (3 Examples)

    A hypothesis test is a formal statistical test we use to reject or fail to reject a statistical hypothesis. We always use the following steps to perform a hypothesis test: Step 1: State the null and alternative hypotheses. The null hypothesis, denoted as H0, is the hypothesis that the sample data occurs purely from chance.

  3. Null Hypothesis: Definition, Rejecting & Examples

    When your sample contains sufficient evidence, you can reject the null and conclude that the effect is statistically significant. Statisticians often denote the null hypothesis as H 0 or H A.. Null Hypothesis H 0: No effect exists in the population.; Alternative Hypothesis H A: The effect exists in the population.; In every study or experiment, researchers assess an effect or relationship.

  4. 6a.1

    The first step in hypothesis testing is to set up two competing hypotheses. The hypotheses are the most important aspect. If the hypotheses are incorrect, your conclusion will also be incorrect. The two hypotheses are named the null hypothesis and the alternative hypothesis. The null hypothesis is typically denoted as H 0.

  5. Hypothesis Testing

    Let's return finally to the question of whether we reject or fail to reject the null hypothesis. If our statistical analysis shows that the significance level is below the cut-off value we have set (e.g., either 0.05 or 0.01), we reject the null hypothesis and accept the alternative hypothesis. Alternatively, if the significance level is above ...

  6. 8.1: The null and alternative hypotheses

    Alternative hypothesis. Alternative hypothesis \(\left(H_{A}\right)\): If we conclude that the null hypothesis is false, or rather and more precisely, we find that we provisionally fail to reject the null hypothesis, then we provisionally accept the alternative hypothesis.The view then is that something other than random chance has influenced the sample observations.

  7. Hypothesis Testing

    There are 5 main steps in hypothesis testing: State your research hypothesis as a null hypothesis and alternate hypothesis (H o) and (H a or H 1). Collect data in a way designed to test the hypothesis. Perform an appropriate statistical test. Decide whether to reject or fail to reject your null hypothesis. Present the findings in your results ...

  8. Hypothesis Testing, P Values, Confidence Intervals, and Significance

    The null hypothesis is deemed true until a study presents significant data to support rejecting the null hypothesis. Based on the results, the investigators will either reject the null hypothesis (if they found significant differences or associations) or fail to reject the null hypothesis (they could not provide proof that there were ...

  9. Better Statistics for Better Decisions: Rejecting Null Hypotheses

    The null hypothesis (H 0) is represented in Figure 2A by a distribution of effects centered on zero; the shaded area represents the probability of sampling an effect d 2 ′ larger than the one obtained ( d 1 ′) if the null hypothesis were true, that is, p (d 2 ′ > d 1 ′ ∣ H 0). This is the conventional "level of significance."

  10. Null & Alternative Hypotheses

    The null hypothesis is the claim that there's no effect in the population. If the sample provides enough evidence against the claim that there's no effect in the population (p ≤ α), then we can reject the null hypothesis. Otherwise, we fail to reject the null hypothesis. Although "fail to reject" may sound awkward, it's the only ...

  11. Understanding Null Hypothesis Testing

    A crucial step in null hypothesis testing is finding the likelihood of the sample result if the null hypothesis were true. This probability is called the p value. A low p value means that the sample result would be unlikely if the null hypothesis were true and leads to the rejection of the null hypothesis. A high p value means that the sample ...

  12. Support or Reject Null Hypothesis in Easy Steps

    Use the P-Value method to support or reject null hypothesis. Step 1: State the null hypothesis and the alternate hypothesis ("the claim"). H o:p ≤ 0.23; H 1:p > 0.23 (claim) Step 2: Compute by dividing the number of positive respondents from the number in the random sample: 63 / 210 = 0.3. Step 3: Find 'p' by converting the stated ...

  13. Why can't we accept the null hypothesis, but we can accept the

    The way that the rules can give confidence about long-run test performance is by specifying what decision applies depending on the data, and the decision relates to the acceptance or non-acceptance (yes, that is rejection as far as I am concerned) of the null hypothesis. Rejection of the statistical null hypothesis can be thought of as ...

  14. S.3.2 Hypothesis Testing (P-Value Approach)

    That is, since the P-value, 0.0127, is less than α = 0.05, we reject the null hypothesis H 0: ... The good news is that, whenever possible, we will take advantage of the test statistics and P-values reported in statistical software, such as Minitab, to conduct our hypothesis tests in this course.

  15. Rejecting the Null Hypothesis Using Confidence Intervals

    As a hypothesis test, we could have the alternative hypothesis as H 1 ≠ 0.51. Since the null value of 0.51 lies within the confidence interval, then we would fail to reject the null hypothesis at ɑ = 0.05. On the other hand, if H 1 ≠ 0.61, then since 0.61 is not in the confidence interval we can reject the null hypothesis at ɑ = 0.05.

  16. Understanding P-Values and Statistical Significance

    In statistical hypothesis testing, you reject the null hypothesis when the p-value is less than or equal to the significance level (α) you set before conducting your test. The significance level is the probability of rejecting the null hypothesis when it is true. Commonly used significance levels are 0.01, 0.05, and 0.10.

  17. Failing to Reject the Null Hypothesis

    Also, does null hypothesis rejecting means whatsoever is stated in null hypothesis that is false proved through regression analysis? ... So, usually the null states there is no effect and you have get good evidence to reject that notion. However, there are a few tests where you actually want to prove something is equal, so you need the null to ...

  18. Understanding the Null Hypothesis for Linear Regression

    x: The value of the predictor variable. Simple linear regression uses the following null and alternative hypotheses: H0: β1 = 0. HA: β1 ≠ 0. The null hypothesis states that the coefficient β1 is equal to zero. In other words, there is no statistically significant relationship between the predictor variable, x, and the response variable, y.

  19. In hypothesis testing why do we need to use the reject null hypothesis

    In hypothesis testing, the common approach is to first set a null hypothesis and a hypothesis we want to test. Then apply some statistical techniques and see whether the observation is likely to happen under the assumption If null hypothesis is true. If the likelihood is low we will then reject the null hypothesis and claim our assumption is true.

  20. Null hypothesis

    The null hypothesis and the alternative hypothesis are types of conjectures used in statistical tests to make statistical inferences, which are formal methods of reaching conclusions and separating scientific claims from statistical noise.. The statement being tested in a test of statistical significance is called the null hypothesis. The test of significance is designed to assess the strength ...

  21. On the logic meaning behind rejecting the null hypothesis and its

    So, what is the logical meaning in rejecting the null-hypothesis when $\alpha$ is viewed as the probability to make a Type I mistake? hypothesis-testing; statistical-significance; p-value; type-i-and-ii-errors; Share. ... but your answer makes good sense to me. Maybe the little that I've studied it is enough. $\endgroup$ - Peter Flom.

  22. Null Hypothesis Definition and Examples

    Null Hypothesis Examples. "Hyperactivity is unrelated to eating sugar " is an example of a null hypothesis. If the hypothesis is tested and found to be false, using statistics, then a connection between hyperactivity and sugar ingestion may be indicated. A significance test is the most common statistical test used to establish confidence in a ...

  23. How to Formulate a Null Hypothesis (With Examples)

    To distinguish it from other hypotheses, the null hypothesis is written as H 0 (which is read as "H-nought," "H-null," or "H-zero"). A significance test is used to determine the likelihood that the results supporting the null hypothesis are not due to chance. A confidence level of 95% or 99% is common. Keep in mind, even if the confidence level is high, there is still a small chance the ...

  24. In Defense of Trying: Why Quitting and Failure are Valuable

    The Freedom of Rejecting "Growth Mindset" In my years of teaching graduate school and practicing therapy , I've seen the impact of this destructive message over and over.